31Дек

Зависимость крутящего момента от мощности и оборотов: гидравлика, гидравлические оборудование, пневматические оборудование, смазочное оборудование, фильтры

Крутящий момент двигателя: что это такое?

В списке ключевых характеристик любого бензинового или дизельного ДВС обязательно указывается мощность и крутящий момент двигателя. Что касается самого транспортного средства, отдельный акцент делается на разгонной динамике автомобиля 0-100 км/ч. независимо от типа силового агрегата под капотом (бензин, дизель, гибридный двигатель и т.д.). Традиционно сложилось, что максимум внимания покупателей изначально обращен на мощность двигателя, выраженную в лошадиных силах (л.с.). Прочно укоренилось мнение, что чем больше л.с. выдает двигатель, тем быстрее, динамичнее и, зачастую, престижнее окажется автомобиль в конечном итоге. Параллельно с этим показатель крутящего момента, который выражается в ньютон-метрах (Н∙м), маркетологи сознательно отодвигают на второй план.

Рекомендуем также прочитать статью о том, что такое объем двигателя. Из этой статьи вы узнаете о том, по каким параметрам измеряется и на что влияет рабочий объем ДВС.

Такой подход хорошо иллюстрирует распространенное выражение среди продавцов автомобилей в США. Как они говорят, продавать машины помогают «лошади», то есть мощность, при этом двигает автомобиль вперед крутящий момент. Далее мы подробно рассмотрим, что такое крутящий момент двигателя внутреннего сгорания, а также взглянем на зависимость характеристик мощности двигателя, крутящего момента и разгонной динамики.

Содержание статьи

  • Мощность и крутящий момент ДВС
  • Как мощность двигателя и крутящий момент влияют на разгон автомобиля
  • Крутящий момент дизельного двигателя
  • Подведем итоги

Мощность и крутящий момент ДВС

Для большинства рядовых автолюбителей понятие о показателе максимальной мощности и крутящего момента сводится к тому, что чем больше мощность, тем больше окажется и крутящего момента, а также более мощный двигатель всегда лучше. При этом чёткое понимание указанных характеристик мотора у многих отсутствует.

Смятение в этот лагерь также внесло растущее число «дизелистов», среди которых намного больше внимания уделяется именно кутящему моменту, а не мощности дизельного мотора. Также следует упомянуть и о турбомоторах, которые могут разгонять автомобиль намного быстрее, хотя мощность самого ДВС с наддувом заметно уступает атмосферным аналогам с намного более внушительным количеством «лошадей» под капотом. Получается, мощнее, но не всегда динамичнее и быстрее? Давайте разбираться, почему так происходит и чем «моментная» характеристика отличается от «мощностной».

Как мощность двигателя и крутящий момент влияют на разгон автомобиля

Как уже было сказано, в технических характеристиках указывается максимальная мощность двигателя и крутящий момент. Итак, крутящий момент представляет собой силу вращения коленвала  ДВС. Измеряется крутящий момент в ньютон-метрах. Также моментная характеристика может быть выражена в килограмм-силах на метр. Крутящий момент возникает тогда, когда свободно вращающийся коленвал начинают тормозить.

Другими словами, на коленвал подается нагрузка, которая заставляет двигать автомобиль.  Отметим, что крутящий момент имеет прямую зависимость от числа оборотов двигателя. Для двигателей внутреннего сгорания характерной особенностью является то, что на низких оборотах крутящий момент небольшой, затем наблюдается рост момента параллельно росту оборотов силового агрегата, далее происходит спад момента, хотя обороты остаются высокими. Обратите внимание, в характеристиках указывается максимальная мощность двигателя, например, 150 л.с. при 6000 об/мин. При этом максимальный крутящий момент указан на отметке 3500-3700 об/мин.

Так происходит по причине того, что на разных оборотах в камере сгорания происходят разные процессы, что отражается на эффективности наполнения цилиндров, качестве сгорания топливно-воздушной смеси, вентиляции цилиндров и т.д. Другими словами, количество воздуха на впуске, угол опережения зажигания, объем отработавших газов и ряд других параметров меняется в зависимости от числа оборотов коленвала. По этой причине каждому водителю бензиновой машины с малообъемным атмосферным мотором хорошо знакома ситуация, когда на «низах» при езде на высокой передаче двигатель не тянет, то есть крутящий момент очень мал.

Нажатие на педаль газа и поднятие оборотов до средних значений приводит к тому, что эффективность наполнения воздухом на впуске растет, топливно-воздушная смесь сгорает более полноценно, цилиндры лучше вентилируются. Результатом становится то, что крутящий момент растет.  Добавим, что турбомоторы в среднем диапазоне оборотов полностью преодолевают эффект турбоямы, после чего у двигателя возникает желаемый подхват. Дело в том, что поток отработавших газов после раскручивания двигателя начинает эффективно вращать крыльчатку турбокомпрессора для подачи большего количества воздуха в цилиндры.

Рекомендуем также прочитать статью об устройстве турбонаддува. Из этой статьи вы узнаете о конструктивных особенностях турбины, а также о преимуществах и недостатках данного способа увеличению мощности двигателя без увеличения его физического объема.

Дальнейший рост оборотов вызывает то, что в двигателе существенно растут механические потери. К таким потерям следует отнести трение поршневых колец о стенки цилиндров, а также различные инерционные потери в других узлах и механизмах двигателя. В результате КПД мотора падает, энергия начинает расходоваться на преодоление таких потерь в условии езды на приближенных к максимальным оборотах.  Закономерно, что крутящий момент начинает уменьшаться с учетом растущих нагрузок. Турбомоторы также теряют отдачу, так как сам турбонагнетатель не обеспечивает должную производительность на максимальных оборотах.

Если сказать иначе, мощность двигателя означает количество работы, которую агрегат способен выполнить за определенный промежуток времени. Мощность ДВС измеряется в киловаттах (кВт) и напрямую зависит от показателя крутящего момента на конкретных оборотах. Не вдаваясь в подробности, мощность является расчетной величиной и не измеряется отдельно от кутящего момента. Что касается максимальной мощности, такая мощность представляет собой условную точку начала уменьшения крутящего момента, но произведение мощности и оборотов еще не стремится к увеличению. С учетом данной информации становится понятно, что такое полка крутящего момента, которая часто отображается на графиках. Под такой полкой следует понимать диапазон оборотов, на которых постоянно доступен максимум крутящего момента.

Простыми словами, крутящий момент и есть мощность двигателя, которая будет доступна на разных оборотах мотора. Этой фактической мощностью, а не разрекламированной маркетологами «максималкой», водители каждый день пользуются во время обгонов и резких ускорений. Вот и получается, что ездим мы на крутящем моменте, а не на максимальной мощности, оценивая динамику разгона на том или ином двигателе. 

Что касается самой максимальной мощности, от данного показателя зависит, прежде всего, та максимальная скорость, с которой способен двигаться автомобиль. Максимальная скорость становится доступной в том случае, когда расходуемая мощность равна мощности ДВС. При этом для определения «максималки» конструкторами учитывается ряд потерь на инерцию и трение, сопротивление потокам воздуха и качению колес. Если проще, от запаса мощности зависит способность мотора преодолевать растущие потери и сопротивление, что и позволяет агрегату разогнать автомобиль только до определенного предела и далее поддерживать набранную скорость.

Крутящий момент дизельного двигателя

Особенностью дизельных двигателей сравнительно с бензиновыми аналогами является более высокий крутящий момент и меньшая мощность. Дело в том, что дизельные моторы имеют суженный диапазон оборотов. Это связано с конструктивными отличиями таких моторов (ход поршня), а также более высокой степенью сжатия и спецификой процесса сгорания дизтоплива.

Другими словами, дизель изначально не приспособлен для работы на высоких оборотах. Следовательно, агрегат не так хорошо раскручивается. Параллельно с этим температура выхлопа у дизельного двигателя ниже по сравнению с бензиновым, а также на «низах» моторы на солярке не так склонны к детонации.  В результате конструкторы смогли установить сложные и максимально эффективные системы турбонаддува именно на дизель.

Благодаря таким особенностям крутящий момент дизельного двигателя на низких оборотах намного выше аналогичных атмосферных или тубированных бензиновых ДВС. Поднимать мощность такого агрегата не имеет смысла, так как уверенная тяга на низах, высокий КПД и топливная экономичность полностью перекрывают небольшое отставание дизелей по показателю мощности и максимальной скорости.

Добавим, что потенциал дизеля позволяет сделать его даже мощнее бензиновых собратьев, но это приведет к существенному удорожанию и утяжелению всей конструкции двигателя. Также понадобится доработка системы питания дизельного мотора и установка более выносливой КПП, которая будет способна выдерживать просто огромный крутящий момент. Не следует забывать и об экологических нормах, для соответствия которым мощные дизели потребуют серьезной модернизации. Получается, поднимать мощность дизеля сегодня попросту нецелесообразно.

Подведем итоги

Если вы столкнулись с возможностью выбрать автомобиль с незначительно отличающимися по характеристикам двигателями, тогда оптимально выбирать агрегат с большим крутящим моментом. Данное правило особенно актуально для машин с МКПП. Например, производитель может выпускать одну и ту же модель, которая получает ДВС с рабочим объемом 1.8 литра (140 л. с.) и 2.0 (155 л.с.). Также следует учитывать и упомянутую выше полку крутящего момента, то есть зависимость мощности и крутящего момента от оборотов двигателя.

Лучшим вариантом двигателя будет тот, когда мотор выходит на пик момента не на определенных оборотах, а в максимально широком диапазоне. Например, простой атмосферный двигатель может иметь пик крутящего момента на 3500 об/мин, в то время как его продвинутый высокотехнологичный аналог с турбиной выходит на пик момента уже при 1500 об/мин, сохраняя «ровную» полку до 4500 об/мин. Это значит, что в первом случае для уверенного разгона мотор нужно крутить, удерживать ДВС на оборотах максимального момента, а также чаще переключать передачи вниз при возникновении нагрузок. Во втором случае максимум крутящего момента будет доступен водителю в широком диапазоне оборотов, что позволяет эффективно ускоряться и справляться с меняющимися нагрузками без частого переключения передачи на пониженную. Другими словами, доступность высокого крутящего момента в расширенном диапазоне фактически означает, что и мощности почти всегда достаточно.

Указанные особенности разных ДВС и умение справляться с нагрузками определяют следующий показатель, который известен как эластичность двигателя. Под эластичностью мотора следует понимать способность агрегата набирать обороты и разгонять автомобиль в условиях растущей нагрузки без переключения передачи на пониженную.

Различные силовые установки тестируются на эластичность путем анализа тяги и разгона с 60 до 100 км/ч при движении на четвёртой передаче или ускорения с 80 до 120 км/ч на включенной пятой передаче. По этой причине малообъемный высокофорсированный двигатель, который имеет отличный подхват на низких оборотах и широкую полку момента, покажет себя отличным вариантом для города. Именно в городском цикле, то есть в условиях умеренных скоростей и режимов ускорение-замедление, потенциала такого ДВС более чем достаточно. При этом следует учитывать, что на более высокой скорости в режиме трассы подобный агрегат может не обеспечить уверенного обгона, уступив в этом плане простому атмосферному двигателю с большим крутящим моментом и мощностью.

Крутящий момент и мощность двигателя особенности и нюансы

Содержание

  • Понятие крутящего момента ДВС. О сложном простыми словами
  • Факторы, влияющие на величину крутящих моментов
  • Влияние крутящего момента ДВС на характеристики автомобиля
  • Крутящий момент vs. мощность. Связь с динамикой автомобиля
  • Дизельный момент. Отличия между КМ бензинового и дизельного двигателей
  • Особенности правильного разгона машины. Как выжать из авто максимум
  • Выбор двигателя. Какой лучше — с высоким моментом или повышенной мощностью?
  • Видео: Мощность и крутящий момент двигателя
  • Видео: Крутящий момент, обороты и мощность двигателя. Простыми словами

Рассуждая о главнейшем автомобильном узле — двигателе, стало принято превозносить мощность превыше других параметров. Между тем, вовсе не мощностные способности являются первостепенной характеристикой силовой установки, а явление, называемое крутящим моментом. Потенциал любого автомобильного двигателя напрямую определяется данной величиной.

 

 

Понятие крутящего момента ДВС. О сложном простыми словами

Крутящим моментом применительно к двигателям автомобилей называется произведение значения силы и плеча рычага, или, простыми словами, сила давления поршня на шатун. Исчисляется эта сила ньютон-метрами, и чем выше ее величина, тем резвее машина.

Более того, мощность двигателя, выражаемая в ваттах, — это не что иное, как умноженное на частоту вращения коленвала значение крутящего момента в ньютон-метрах.

Представим лошадь, которая тащит тяжелые сани и увязает в канаве. Вытянуть сани не получится, если лошадь будет пытаться выскочить из канавы с разбега. Здесь необходимо приложить определенную силу, которая и будет являться крутящим моментом (КМ).

Часто крутящий момент путают с частотой вращения коленвала. В реальности это два совершенно разных понятия. Если вернуться к примеру с лошадью, застрявшей в канаве, частота шага будет символизировать частоту оборотов двигателя, тогда как сила, прикладываемая животным при отталкивании во время шага, олицетворяет в данном случае крутящий момент.

Факторы, влияющие на величину крутящих моментов

Из примера с лошадью легко догадаться, что в данном случае значение КМ будет во многом определяться мышечной массой животного. Применительно к автомобильному двигателю внутреннего сгорания эта величина зависит от рабочего объема силовой установки, а также от:

  • уровня рабочего давления внутри цилиндров;
  • размера поршня;
  • диаметра кривошипа коленвала.

 

Наиболее сильно крутящий момент зависим от рабочего объема и давления внутри силовой установки, и эта зависимость прямо пропорциональна. Другими словами, двигатели с большим объемом и давлением, соответственно, отличаются и большим моментом.

Прямая зависимость наблюдается также между КМ и радиусом кривошипа коленвала. Однако конструкция современных автомобильных двигателей такова, что не позволяет варьировать значения момента в широких пределах, из-за чего возможности добиться повышенного крутящего момента за счет радиуса кривошипа коленчатого вала у конструкторов ДВС невелики. Вместо этого разработчики прибегают к таким способам увеличить момент, как использование технологий турбонаддува, увеличение степени сжатия, оптимизация процесса сгорания топлива, использование впускных коллекторов специальных конструкций, и т.д.

Важно, что КМ увеличивается с ростом оборотов двигателя, однако после достижения максимума на определенном диапазоне крутящий момент понижается несмотря на продолжающийся прирост частоты вращения коленвала.

 

 

Влияние крутящего момента ДВС на характеристики автомобиля

Величина крутящего момента выступает тем самым фактором, который непосредственным образом задает динамику разгона автомобиля. Если вы — заядлый автолюбитель, то могли заметить, что разные автомобили, но с одинаковым силовым агрегатом, по-разному ведут себя на дороге. Или на порядок менее мощный автомобиль на дороге превосходит того, у которого под капотом лошадиных сил больше, причем даже тогда, когда сравнимые авто имеют одинаковые размеры и вес. Причина заключается как раз в разнице в крутящих моментах.

Лошадиные силы можно представить как индикатор выносливости мотора. Именно этот показатель определяет скоростные возможности автомобиля. Но поскольку крутящий момент является разновидностью силы, то непосредственно от его величины, а не от количества «лошадей», зависит то, насколько быстро автомобиль сможет достичь максимального скоростного режима. По этой причине далеко не каждое мощное авто обладает хорошей динамикой разгона, а те, что способны разгоняться быстрее других, необязательно оснащены мощным двигателем.

Вместе с тем высокий крутящий момент еще не гарантирует сам по себе отличную динамичность машины. Ведь кроме прочего, динамика увеличения скорости, а также способность авто к резвому преодолению подъемов участков, зависит от диапазона работы силовой установки, передаточных чисел трансмиссии, отзывчивости педали газа. Наряду с этим нужно учитывать, что момент существенно понижается из-за различных противодействующих явлений — сил качения колес и трения в различных автомобильных узлах, из-за аэродинамических и прочих явлений.

Крутящий момент vs. мощность. Связь с динамикой автомобиля

Мощность — производное такого явления, как крутящий момент, ею выражается работа силовой установки, выполненная за определенное время. А поскольку КМ олицетворяет собой непосредственную работу мотора, то в виде мощности отражается величина момента в соответствующий период времени.

Наглядно увидеть связь между мощностью и КМ позволяет следующая формула:

P=М*N/9549

 

Где: P в формуле означает мощность, М — крутящий момент, N — обороты двигателя за минуту, а 9549 — коэффициент обращения N в радианы в секунды. Результатом вычислений по данной формуле будет являться число в киловаттах. Когда нужно перевести полученный результат в лошадиные силы, полученное число умножают на 1. 36.

По сути, крутящим моментом является мощность при неполных оборотах, например, во время обгона. Мощность возрастает по мере роста момента, и чем выше этот параметр, тем больше запас кинетической энергии, тем легче автомобиль преодолевает противодействующие на него силы и тем лучше его динамические характеристики.

При этом важно помнить, что мощность достигает своих максимальных значений не сразу, а постепенно. Ведь с места автомобиль трогается на минимуме оборотов, и затем скорость наращивается. Именно здесь и подключается сила под названием крутящий момент, и именно она определяет тот самый временной отрезок, за который авто достигнет своей пиковой мощности, или, другими словами, скоростную динамику.

 

 

Из этого следует, что машина с силовым агрегатом мощнее, но обладающим недостаточно высоким крутящим моментом, уступит по скорости разгона модели с мотором, который, напротив, не способен похвастать хорошей мощностью, но превосходит конкурента в крутящем моменте. Чем большая тяга, сила передается ведущим колесам и чем богаче диапазон оборотов силовой установки, в котором достигается высокий КМ, тем быстрее происходит ускорение автомобиля.

В то же время существование крутящего момента возможно без мощности, но существование мощности без момента — нет. Представьте, что наша лошадь с санями увязла в грязи. Производимая лошадью мощность в этот момент будет равняться нулю, но крутящий момент (попытки выбраться, тяга), хотя его может быть недостаточно для движения, будет присутствовать.

 

Дизельный момент. Отличия между КМ бензинового и дизельного двигателей

Если сравнивать бензиновые силовые установки с дизельными, то отличительной особенностью последних (всех без исключения) является повышенный крутящий момент при меньшем количестве лошадиных сил.

Бензиновый ДВС достигает своих максимальных значений КМ при трех-четырех тысячах оборотов в минуту, но затем способен стремительно нарастить мощность, раскрутившись за минуту до семи-восьми тысяч раз. Диапазон оборотов же коленчатого вала дизельного двигателя обычно ограничен тремя-пятью тысячами. Однако в дизельных установках больше ход поршня, выше уровень сжатия и другая специфика сгорания топлива, что обеспечивает не только более высокий относительно бензиновых установок крутящий момент, но и доступность этой силы едва ли не с холостого хода.

По этой причине смысла добиваться повышенной мощности дизельных двигателей нет: уверенная, доступная «с низов» тяга, высокий коэффициент полезного действия и топливная эффективность полностью нивелируют отставание таких ДВС от бензиновых как по мощностным показателям, так и по скоростному потенциалу.

Особенности правильного разгона машины. Как выжать из авто максимум

Основа правильного разгона — умение работать с коробкой передач и следование принципу «от максимума момента до пика мощности». То есть, добиться наилучшей динамики разгона машины можно только поддерживая частоту вращения коленвала в том диапазоне значений, при которых КМ достигает своего максимума. Очень важно, чтобы обороты совпали с пиком крутящего момента, но при этом должен оставаться запас по их увеличению. Если разгоняться на оборотах выше пиковой мощности, динамика разгона будет меньше.

Диапазон оборотов, соответствующий максимуму крутящего момента, обусловлен характеристиками двигателя.

Выбор двигателя. Какой лучше — с высоким моментом или повышенной мощностью?

Если подвести итоговую черту под всем вышесказанным, то станет очевидно, что:

  • крутящий момент — ключевой фактор, характеризующий возможности силовой установки;
  • мощность — это производная КМ и, соответственно, вторичная характеристика двигателя;
  • прямую зависимость мощности от момента можно увидеть по выведенной физиками формуле Р (мощность) = М (момент) * n (частота вращения коленвала в минуту).

Таким образом, выбирая между двигателем с большим количеством лошадиных сил, но меньшим крутящим моментом, и двигателем с большим КМ, но меньшей мощностью, приоритетным будет второй вариант. Использовать весь заложенный в автомобиль потенциал позволит только такой мотор.

При этом не следует забывать о взаимосвязи динамических характеристик автомобиля с такими факторами, как отзывчивость педали газа и коробка переключения передач. Лучшим вариантом станет то авто, которое не только оснащено двигателем с высоким крутящим моментом, но и имеет наименьшую длину задержки между нажатием педали газа и реакцией двигателя, а также трансмиссию с короткими соотношениями передач. Наличие этих особенностей компенсирует маломощность силовой установки, заставляя автомобиль разгоняться быстрее, чем машина с двигателем похожей конструкции, но с меньшей силой тяги.

Видео: Мощность и крутящий момент двигателя

Видео: Крутящий момент, обороты и мощность двигателя. Простыми словами

Мощность через момент и обороты формула

Мощность двигателя – это величина, показывающая, какую работу способен совершить мотор в единицу времени. То есть то количество энергии, которую двигатель передает на трансмиссию за определенный временной промежуток. Измеряется в киловаттах (кВт) или лошадиных силах (л. с.).

Как рассчитывается мощность двигателя?

Расчет мощности мотора проводится несколькими способами. Самый доступный способ – через крутящий момент. Умножаем крутящий момент на угловую скорость – получаем мощность двигателя.

N_дв=M∙ω=2∙π∙M∙n_дв

N_дв – мощность двигателя, кВт;

M – крутящий момент, Нм;

ω – угловая скорость вращения коленчатого вала, рад/сек;

π – математическая постоянная, равная 3,14;

n_дв – частота вращения двигателя, мин-1.

Мощность рассчитывается и через среднее эффективное давление. Камера сгорания имеет определенный объем. Разогретые газы воздействуют на поршень в цилиндре с определенным давлением. Двигатель вращается с некоторой частотой. Произведение объема двигателя, среднего эффективного давления и частоты вращения, поделенное на 120, и даст теоретическую мощность двигателя в кВт.

N_дв=(V_дв∙P_эфф∙n_дв)/120

V_дв – объем двигателя, см3;

P_эфф – эффективное давление в цилиндрах, МПа;

120 – коэффициент, применяемый для расчета мощности четырехтактного двигателя (у двухтактных ДВС этот коэффициент равен 60).

Для расчета лошадиных сил киловатты умножаем на 0,74.

N_(дв л.с.)=N_дв∙0,74

N_дв л.с. – мощность двигателя в лошадиных силах, л. с.

Другие формулы мощности двигателя используются в реальных расчетах реже. Эти формулы включают в себя специфичные переменные. И чтобы измерить мощность двигателя по другим методикам, нужно знать производительность форсунок или массу потребленного двигателем воздуха.

На практике расчет мощности автопроизводители выполняют эмпирическим способом, то есть замеряют на стенде и строят график зависимости по факту, на основании полученных во время испытаний показателей.

Мощность двигателя – величина непостоянная. Для каждого мотора есть кривая, которая отображает на графике зависимость мощности от частоты вращения коленчатого вала. До определенного пика, примерно до 4-5 тысяч оборотов, мощность растет пропорционально оборотам. Далее идет плавное отставание роста мощности, кривая наклоняется. Примерно к 7-8 тысячам оборотов мощность идет на спад. Сказывается перекрытие клапанов на большой частоте вращения коленвала и падение КПД мотора из-за недостаточно интенсивного газообмена.

Чтобы узнать мощность двигателя, обратитесь к инструкции по эксплуатации авто. В разделе с техническими характеристиками мотора будет указана мощность и обороты, при которых она достигает пикового значения. Если мощность указана киловаттах, чтобы рассчитать лошадиные силы двигателя, воспользуйтесь приведенной выше формулой. В некоторых случаях автопроизводитель предоставляет график, на котором есть зависимость мощности двигателя и крутящего момента от частоты оборотов.

Видео: Простыми словами без сложных формул и расчетов, что такое мощность, крутящий момент и обороты двигателя.

Мощность ДВС определяет, насколько быстро автомобиль способен передвигаться или ускоряться (совершать работу). Полезная мощность двигателя рассчитывается с учетом потерь в трансмиссии, то есть указывает, сколько от изначальной мощности мотора по факту доходит до колес авто.

Что такое крутящий момент

Крутящий момент в двигателе автомобиля – это вращающая сила, которая численно равна произведению приложенной силы (давление раскаленных газов на поршень) на плечо (расстояние между осями коренных и шатунных шеек коленчатого вала в проекции, перпендикулярной оси вращения коленвала). Измеряется крутящий момент в ньютонах на метр (Нм).

Крутящий момент ДВС зависит от силы давления на поршень и расстояния между коренными и шатунными шейками. Зависимость здесь прямая. Чем больше плечо и чем больше давление на поршень – тем больше крутящий момент двигателя.

У дизельных двигателей степень сжатия больше. Больше и ход поршня в цилиндре (при равном с бензиновым мотором диаметре цилиндров). А это значит, что и расстояние между коренными и шатунными шейками будет больше. То есть длиннее плечо. За счет большей степени сжатия при рабочем такте у дизелей выше сила, давящая на поршень. Крутящий момент в дизельных моторах при прочих равных больше, чем в бензиновых.

Крутящий момент влияет на то, сколько энергии отдает мотор в текущий момент времени. Крутящий момент есть та величина, которая определяет фактически передаваемую в данный момент времени энергию на трансмиссию. Чем больше момент, тем сильнее тяга двигателя при текущих оборотах.

Что лучше: мощность или крутящий момент

Мощность и крутящий момент двигателя – величины взаимосвязанные. Это хорошо видно в формуле из первого пункта.

Пик крутящего момента на графике зависимости от частоты вращения мотора появляется раньше, чем пик мощности. Это справедливо как для дизельных, так и для бензиновых моторов. Однако у дизелей крутящий момент достигается раньше, и плато (интервал частоты вращения при пиковом значении) длиннее. У бензиновых ДВС мощность выше, хотя для ее достижения нужно раскрутить мотор почти до максимальных оборотов.

Сказать определенно, что лучше: мощность или крутящий момент, нельзя. Все зависит от случая. Трансмиссия современного авто способна трансформировать эти величины под требуемые условия. Поясним на примерах.

Для тяжелой техники, которой важна тяга в широком диапазоне оборотов, важнее крутящий момент. Мотор должен хорошо тянуть. Раскручивать его до предельных оборотов не нужно. Отчасти поэтому почти вся коммерческая техника оснащается дизельными моторами.

В гоночных автомобилях важнее мощность. Моторы этих авто по оборотам пилоты во время заездов держат в красной зоне. Двигатель отдает максимальную мощность. А трансмиссия преобразовывает мощность в тягу.

Для гражданских авто важен стиль вождения. Для езды на автомате подойдут оба мотора. Автоматическая трансмиссия будет держать мотор в диапазоне оборотов, при которых двигатель отдает максимум своего потенциала.

Для агрессивной езды на механике с раскручиванием двигателя в красную зону тахометра лучше подойдет бензиновый мотор. Но в этом случае нужно понимать, что для получения максимальной производительности от мотора потребуется держать его на пике оборотов и часто переключать передачи. Пик мощности у бензинового ДВС имеет малый диапазон и находится около максимальных оборотов. Для уверенных обгонов и ускорений нужно будет понижать передачу и раскручивать двигатель.

Для размеренной езды, особенно в городе, больше подходит дизель. Для обгона на дизельном авто зачастую не потребуется переходить на пониженную передачу, а высокий крутящий момент в широком диапазоне оборотов позволит реже переключаться.

Материал подготовлен автором проекта АвтобурУм. Графики можно увидеть здесь: https://autoburum.com/user/stas90/blog/609-moshhnost-dvigate.

Большинство автолюбителей судят о ходовых характеристиках авто по мощности двигателя. Обычно ее измеряют в киловаттах или лошадиных силах. Чем она больше, тем солиднее. Максимальную мощность двигатель внутреннего сгорания развивает на определенных оборотах. Обычно для бензиновых автомобилей это около 6000 оборотов в минуту, для дизельных – около 4000 об./мин. Именно поэтому дизельные движки относятся к классу низкооборотных, бензиновые – высокооборотные. Однако и среди бензиновых двигателей есть низкооборотные, и наоборот – есть дизельные высокооборотные.

Часто водитель сталкивается с ситуацией, когда необходимо придать авто значительное ускорение для выполнения очередного маневра. Жмешь педалью акселератора в пол, а автомобиль практически не ускоряется. Вот тут-то и нужен мощный крутящий момент на тех оборотах, на которых работает в данный момент двигатель. Именно он характеризует приемистость автомобиля. Поэтому каждый автовладелец должен знать, на каких оборотах его авто имеет максимальный крутящий момент перед тем, как садить красивую девушку в свою машину и показывать чудеса пилотирования.

Крутящий момент двигателя, что это?

Из курса физики за 9 класс многие помнят, что крутящий момент М равен произведению силы F, прикладываемой к рычагу длиной плеча L. Формула:

Длина в системе СИ измеряется в метрах, сила – в ньютонах. Нетрудно определить, что момент измеряется в ньютон на метр.

Основная сила в двигателе внутреннего сгорания вырабатывается в камере сгорания в момент воспламенения смеси. Она приводит в действие кривошипно-шатунный механизм коленвала. Рычагом здесь является длина кривошипа, то есть, если эта длина будет больше, то и крутящий момент тоже увеличивается. Однако, увеличивать кривошипный рычаг бесконечно нельзя. Во-первых, тогда надо увеличивать рабочий ход поршня, то есть размеры движка. Во-вторых, при этом уменьшаются обороты двигателя. Двигатели с большим рычагом кривошипного механизма применяют в крупномерных плавательных средствах. В легковых авто с небольшими размерами коленвала не поэкспериментируешь.

В технических характеристиках, указанных на модель двигателя, параметр максимального крутящего момента указывается совместно с величиной оборотов (либо пределами величин оборотов), при которых такой крутящий момент может быть достигнут. Обычно считается: если максимальный крутящий момент может быть достигнут на оборотах до 4500 об./мин., то двигатель низкооборотный, более 4500 – высокооборотный.

От величины крутящего момента напрямую зависит характеристика мощности двигателя автомобиля. Почему считается, что бензиновые движки заведомо могут обеспечить большую, чем дизельные, мощность. Дело в том, что в силу конструктивных особенностей и управляемости системы зажигания бензиновые двигатели могут длительное время работать на оборотах 8000 об./мин и более. Дизельные движки достигают максимального крутящего момента на более низких оборотах. В городском ритме движения, когда нет необходимости развивать предельные обороты, дизельные авто нисколько не уступают бензиновым, наоборот, на малых и средних оборотах спокойно можно двигаться в ритме от 30 до 60 км/час, не переключая третью либо 4-ю передачу.

Пересчитать крутящий момент в мощность двигателя и наоборот можно, руководствуясь упрощенной физической формулой:

По этой формуле получится мощность Р в киловаттах. Вводить надо М – крутящий момент двигателя в ньютон на метр, n– величина оборотов двигателя. Здесь 9549 — число, которое получается после упрощения основной формулы в результате перемножения констант (ускорения свободного падения, числа Пи и т. п.).

Для перевода киловатт в лошадиные силы следует результат умножить на 1,36. В некоторых случаях в технических характеристиках указывается крутящий момент на холостых оборотах.

Зависимости мощности двигателя и крутящего момента от количества оборотов

Типовые характеристики зависимости мощности и крутящего момента от оборотов двигателя приведены на рис.1

Из графика видно, что крутящий момент стабильно увеличивается до 3000 оборотов, затем наступает относительно пологий участок. На оборотах около 4500 об/мин достигается максимум крутящего момента около 178 ньютон*метр. В то же время мощность двигателя продолжает расти до достижения оборотов около 5500 об/мин, и на этих оборотах достигает около 124 лошадиных сил. Это понятно, если обратиться к формуле, в которой видно, что мощность пропорциональна произведению крутящего момента на величину оборотов. После 5500 оборотов в минуту уменьшение крутящего момента превышает крутизну увеличения оборотов, и мощность начинает уменьшаться.

Как это объяснить физически, то есть, без формул. На малых оборотах в область сгорания поступает небольшое количество воздушно-топливной смеси в единицу времени, соответственно, крутящий момент и мощность небольшие. Увеличивая обороты, количество смеси (а вслед за ним и мощность, крутящий момент) возрастает. Достигая больших значений, мощность уменьшается по следующим причинам:

механические потери на трение механизмов;

недостаточное нагнетание воздуха (кислородное голодание).

Из соображений обеспечения максимального количества поступающего воздуха (кислорода) в камеру сгорания даже на небольших оборотах двигателя применяют системы турбонаддува с электронным регулированием. Используя такие системы можно обеспечить равномерность характеристик крутящего момента в широком диапазоне оборотов двигателя, как показано на рис.2

Уровень максимального крутящего момента около 242 ньютон на метр поддерживается в пределах от 2000 до 5000 об/мин коленвала. Это значит, что можно без волнений начинать обгон, двигаясь на относительно низких оборотах двигателя.

Высокооборотные движки позволяют максимально увеличивать мощность за счет уверенной работы на предельно высоких оборотах вплоть да 8000 об/мин, как показано на рис.3

Если вы серьезно подходите к динамическим характеристикам своего или вновь приобретаемого автомобиля, знать характеристики крутящего момента и мощности двигателя в зависимости от оборотов просто необходимо. Их можно найти, покопавшись на различных форумах, сайтах автодилеров и производителей.

Для городского ритма движения лучше подойдут низкооборотные двигатели с турбонаддувом. Если вы любите попалить резину, посоперничать на трассе, лучше выбрать автомобиль с высокооборотным бензиновым движком.

Можно ли увеличить крутящий момент двигателя

Величину необходимого крутящего момента определяют конструкторы еще на предварительном этапе конструкторской разработки двигателя внутреннего сгорания. От нее зависят и другие элементы автомобиля: подвеска, тормозная и рулевая система, аэродинамика. Поэтому, прежде чем приступить к самостоятельному форсированию двигателя, убедитесь, что ваша машина не развалится или не улетит в космос на умощненном двигателе.

Способов увеличения крутящего момента и, соответственно, мощности много:

изменение геометрических свойств поршневой группы, увеличение компрессии;

замена форсунок или инжекторов;

внесение изменений в систему воздухозабора;

чип-тюнинг путем перепрограммирования топливной карты блока управления двигателя.

Опыт показывает, что принудительное увеличение крутящего момента и мощности двигателя на 20% уменьшает ресурс его работы приблизительно в два раза. Поэтому, если вы не фанат дрэг-рейсинга, дрифтинга и красивых девушек, лучше не экспериментировать.

Этот калькулятор позволяет перевести мощность и момент силы и обратно для заданной угловой скорости

Ниже два калькулятора, которые переводят мощность в момент силы (или крутящий момент) и наоборот для заданной угловой скорости. Формулы под калькулятором.

Момент силы и мощность

Мощность и момент силы

Несколько формул/
Для мощности:

где P — мощность (Ватты или килоВатты), τ — крутящий момент (Ньютон-метр), ω — угловая скорость (радиан в секунду), а точка обозначает скалярное произведение.
Для момента силы:

Угловая скорость в калькуляторе задается в оборотах в минуту, приведение ее к радианам в секунду тривиально:

Формула расчета крутящего момента

Мощность двигателя – это величина, показывающая, какую работу способен совершить мотор в единицу времени. То есть то количество энергии, которую двигатель передает на трансмиссию за определенный временной промежуток. Измеряется в киловаттах (кВт) или лошадиных силах (л. с.).

Как рассчитывается мощность двигателя?

Расчет мощности мотора проводится несколькими способами. Самый доступный способ – через крутящий момент. Умножаем крутящий момент на угловую скорость – получаем мощность двигателя.

N_дв=M∙ω=2∙π∙M∙n_дв

N_дв – мощность двигателя, кВт;

M – крутящий момент, Нм;

ω – угловая скорость вращения коленчатого вала, рад/сек;

π – математическая постоянная, равная 3,14;

n_дв – частота вращения двигателя, мин-1.

Мощность рассчитывается и через среднее эффективное давление. Камера сгорания имеет определенный объем. Разогретые газы воздействуют на поршень в цилиндре с определенным давлением. Двигатель вращается с некоторой частотой. Произведение объема двигателя, среднего эффективного давления и частоты вращения, поделенное на 120, и даст теоретическую мощность двигателя в кВт.

N_дв=(V_дв∙P_эфф∙n_дв)/120

V_дв – объем двигателя, см3;

P_эфф – эффективное давление в цилиндрах, МПа;

120 – коэффициент, применяемый для расчета мощности четырехтактного двигателя (у двухтактных ДВС этот коэффициент равен 60).

Для расчета лошадиных сил киловатты умножаем на 0,74.

N_(дв л.с.)=N_дв∙0,74

N_дв л.с. – мощность двигателя в лошадиных силах, л. с.

Другие формулы мощности двигателя используются в реальных расчетах реже. Эти формулы включают в себя специфичные переменные. И чтобы измерить мощность двигателя по другим методикам, нужно знать производительность форсунок или массу потребленного двигателем воздуха.

На практике расчет мощности автопроизводители выполняют эмпирическим способом, то есть замеряют на стенде и строят график зависимости по факту, на основании полученных во время испытаний показателей.

Мощность двигателя – величина непостоянная. Для каждого мотора есть кривая, которая отображает на графике зависимость мощности от частоты вращения коленчатого вала. До определенного пика, примерно до 4-5 тысяч оборотов, мощность растет пропорционально оборотам. Далее идет плавное отставание роста мощности, кривая наклоняется. Примерно к 7-8 тысячам оборотов мощность идет на спад. Сказывается перекрытие клапанов на большой частоте вращения коленвала и падение КПД мотора из-за недостаточно интенсивного газообмена.

Чтобы узнать мощность двигателя, обратитесь к инструкции по эксплуатации авто. В разделе с техническими характеристиками мотора будет указана мощность и обороты, при которых она достигает пикового значения. Если мощность указана киловаттах, чтобы рассчитать лошадиные силы двигателя, воспользуйтесь приведенной выше формулой. В некоторых случаях автопроизводитель предоставляет график, на котором есть зависимость мощности двигателя и крутящего момента от частоты оборотов.

Видео: Простыми словами без сложных формул и расчетов, что такое мощность, крутящий момент и обороты двигателя.

Мощность ДВС определяет, насколько быстро автомобиль способен передвигаться или ускоряться (совершать работу). Полезная мощность двигателя рассчитывается с учетом потерь в трансмиссии, то есть указывает, сколько от изначальной мощности мотора по факту доходит до колес авто.

Что такое крутящий момент

Крутящий момент в двигателе автомобиля – это вращающая сила, которая численно равна произведению приложенной силы (давление раскаленных газов на поршень) на плечо (расстояние между осями коренных и шатунных шеек коленчатого вала в проекции, перпендикулярной оси вращения коленвала). Измеряется крутящий момент в ньютонах на метр (Нм).

Крутящий момент ДВС зависит от силы давления на поршень и расстояния между коренными и шатунными шейками. Зависимость здесь прямая. Чем больше плечо и чем больше давление на поршень – тем больше крутящий момент двигателя.

У дизельных двигателей степень сжатия больше. Больше и ход поршня в цилиндре (при равном с бензиновым мотором диаметре цилиндров). А это значит, что и расстояние между коренными и шатунными шейками будет больше. То есть длиннее плечо. За счет большей степени сжатия при рабочем такте у дизелей выше сила, давящая на поршень. Крутящий момент в дизельных моторах при прочих равных больше, чем в бензиновых.

Крутящий момент влияет на то, сколько энергии отдает мотор в текущий момент времени. Крутящий момент есть та величина, которая определяет фактически передаваемую в данный момент времени энергию на трансмиссию. Чем больше момент, тем сильнее тяга двигателя при текущих оборотах.

Что лучше: мощность или крутящий момент

Мощность и крутящий момент двигателя – величины взаимосвязанные. Это хорошо видно в формуле из первого пункта.

Пик крутящего момента на графике зависимости от частоты вращения мотора появляется раньше, чем пик мощности. Это справедливо как для дизельных, так и для бензиновых моторов. Однако у дизелей крутящий момент достигается раньше, и плато (интервал частоты вращения при пиковом значении) длиннее. У бензиновых ДВС мощность выше, хотя для ее достижения нужно раскрутить мотор почти до максимальных оборотов.

Сказать определенно, что лучше: мощность или крутящий момент, нельзя. Все зависит от случая. Трансмиссия современного авто способна трансформировать эти величины под требуемые условия. Поясним на примерах.

Для тяжелой техники, которой важна тяга в широком диапазоне оборотов, важнее крутящий момент. Мотор должен хорошо тянуть. Раскручивать его до предельных оборотов не нужно. Отчасти поэтому почти вся коммерческая техника оснащается дизельными моторами.

В гоночных автомобилях важнее мощность. Моторы этих авто по оборотам пилоты во время заездов держат в красной зоне. Двигатель отдает максимальную мощность. А трансмиссия преобразовывает мощность в тягу.

Для гражданских авто важен стиль вождения. Для езды на автомате подойдут оба мотора. Автоматическая трансмиссия будет держать мотор в диапазоне оборотов, при которых двигатель отдает максимум своего потенциала.

Для агрессивной езды на механике с раскручиванием двигателя в красную зону тахометра лучше подойдет бензиновый мотор. Но в этом случае нужно понимать, что для получения максимальной производительности от мотора потребуется держать его на пике оборотов и часто переключать передачи. Пик мощности у бензинового ДВС имеет малый диапазон и находится около максимальных оборотов. Для уверенных обгонов и ускорений нужно будет понижать передачу и раскручивать двигатель.

Для размеренной езды, особенно в городе, больше подходит дизель. Для обгона на дизельном авто зачастую не потребуется переходить на пониженную передачу, а высокий крутящий момент в широком диапазоне оборотов позволит реже переключаться.

Как известно — стенды измеряют момент с колёс, (или все-таки мощность напрямую?) а мощность получается путём пересчёта.
Вот нашёл такую формулу, теперь каждый может проверить свой график.

Формула расчета мощности в зависимости от крутящего момента и оборотов двигателя:

P = Mкр х N : 9549, где:

Р — мощность в кВт (кило Ваттах)
Mкр — крутящий момент в Hм (Ньютона метрах)
N — обороты мотора об/мин
9549- это коэффициент, что бы не возится с косинусами альфа и обороты подставлять в об/мин.

Например, если мотор выдает 357 Нм момента при 4400 об/мин, его мощность в киловаттах:
357 x 4400 : 9549 =164,5 (кВт)

164.5 х 1.36 = 223,72 л.с.

Я свой график проверил))

PS: Так я прав: Первично на барабанах снимают момент в Нм?
А силы получают расчётом?

Комментарии 55

стенд измеряет мощность, а момент высчитывает

Что измеряет омметр?
Правильно, сопротивление.
Но на самом деле, чтобы измерить сопротивление, нужно приложить к измеряемому объекту напряжение, измерить ток, протекающий через этот объект, затем поделить напряжение на ток.
Так что измеряет омметр?
ТОК!
А сопротивление он вычисляет.

Мощность — это работа, совершенная за какой-то промежуток времени.
Мы можем измерить мощность?
Мы можем измерить произведенную работу,
и поделить ее на время.

Однажды заехал электромобиль. У него тахометра с оборотами как такового нету. Разьема ОБД нету. Но есть спидометр. Синхронизацию делали по спидометру (по соответствию скорости). Ну какая там точность?

В принципе момент можно нарисовать ЛЮБОЙ. Т.к. сделав неточность (специально завышенную или заниженную по величине синхронизацию) выставления оборотов-скорости ВЫ получите, что пожелаете!

А значит стендом мерится НЕ МОМЕНТ … а мощность

Есть мощность и момент КОЛЕСНЫЕ. В сумме с механическими потерями получается мощность МОТОРНАЯ.
Но что первично мерится МОМЕНТ или МОЩНОСТЬ? (переписываю ВАш вопрос).

Логика говорит, что момент первичен и уже потом пересчет в мощность. Однако наблюдая за работой колесного стенда картина выглядит наоборот…т. е.
— В большинстве случаев операторы делают синхранизацию скорости вращения роликов стенда с оборотами замеряемого ДВИГАТЕЛЯ. У стендов есть такая опция.
Измерение оборотов можно сделать и традиционными методами, скажем прищепкой индукции прикрепленной на высоковольтный провод или оптическим методом от вращения шкиваколенвала и т.п.

Однако в большинстве используют именно синхронизацию вращения роликов с оборотами ДВС.

Предположим мы вообще не делаем синхранизацию. Т.е. стенду быдут передаваться лживые обороты двигателя. Делая замер мотора фольцваген ПОЛО 1,6 литра (атмосферный), Вы получите примерно следующие цифры…

Что по мощности соответствует с реалиями, а по моменту нет! Точность выставления оборотов ДВС и скорости вращения роликов ВАЖНА. НЕ точность приводит и лживому расчету (преесчету) момента.

Но мощность ВСЕГДА верная! А момент может быть разный. Исходя из этого я утверждаю, что замеряется мощность, а момент ПЕРЕСЧИТЫВАЕТСЯ из данных мощности.

вот и реальная картинка замера без синхронизации оборотов двс и скорости роликов. forum.bratsk.org/attachme…ent >

Все верно, стенду вообще ничего не нужно знать об измеряемом а/м (кроме типа привода), он измерит мощность на своих барабанах (или ступицах) и получим первичный график мощности от линейной скорости точки соприкосновения колеса и барабана.
А вот чтобы получить крутящий момент двигателя с потерями в трансмиссии и т.п., нужно уже знать обороты двигателя, т.е. привязку оборотов двигателя к скорости колеса.

Стенды измеряют МОЩНОСТЬ, причем с колёс. Моторная мощность и момент — расчётные величины.

А alexkolomna утверждает обратное)

Я видел.
Вот тебе много букв и много формул: www.drive2.ru/b/2914486/

Это один из текстов, который я почти на изусть знаю)))

А вот ключевые слова именно про инерционный стенд ( читата из текста по ссылке))))
Получается, что все нужное для определения мощности есть – момент (МОМЕНТ !) инерции известен, угловую скорость измеряет датчик вращения на оси, время может отсекать управляющий компьютер.

Так ты сам всё притягиваешь к «курица-яйцо». Стенд занимается измерением мощности с колёс, зная константы. Далее все остальные расчёты.

Момент первичен)
И только потом мощность.
Если не верно указать момент энерции стенда — все остальные расчеты пойдут прахом

Холивара не будет, мы разговариваем на разных языках, я лучше сольюсь.

Так ты сам всё притягиваешь к «курица-яйцо». Стенд занимается измерением мощности с колёс, зная константы. Далее все остальные расчёты.

Я всегда думал что стенд сделан по уму… а тут

Вот если бы на стенде стоял генератор и нагрузка: резистор не хилый. Расчетные и практические КПД генератора как константа и вот вам чистая мощность… ток умножаем на напряжение. Нагружаем мотор как хотим Этим резистором.

Да стенд немного усложняется зато киловат в киловат покажет а дальше можно считать и момент и все остальное относительно оборотов движка снятых с диагностического шнура.

Значительно проще и не менее точно посчитает инерционный стенд, с достаточной массой подвижной системы. И не нужно ничего выдумывать.
Предложенная тобой схема, на сколько мне известно, не используется, предположу, что проблемы будут с надежностью и температурной стабильностью. К тому же это не решает самой главной проблемы: стенд по прежнему продолжает измерять мощность с колёс.

Температурная стабильность и все дела все это фигня — печка на 200 кВт пару минут поработает. Даже на пару градусов температура в гараже не поднимится.

Сколес мериет мощность — а она так сильно отличается от моторной? потери на трансмиссии минимальные. Да и какая радость от мощности мотора если фактически используется мощность с колес.

Инерционный стенд в совокупности с колесами взаимодействует а инерционность колес как раз и вносит погрешность измерений. А при измерении тепловой мощности — никакие инерции не внесут погрешность.

Нравится мне, как за 20 минут, тыкая в кнопки клавиатуры, один человек ломает привычную всем картину.
Но не буду я и с тобой спорить. Собери правильный стенд и докажи всем производителям подобного оборудования, а за одно и нам, что твоя теория верна, а все вокруг заблуждались.

Ага мне тоже нравится за 20 минут разломать привычную картину )))

Я не говорю что все вокруг заблуждаются — просто надо понимать что он может измерить имея такой принцип работы.

Давно подумывал собрать сненд такого типа как описал выше, тока генератор дороговат получится.

С законами физики не поспоришь что тут сделать.

Ища инфу натыкаюсь на интересные факты. когда потери на трансмисии достигают 10%-20% — это мотор крутанул до 100 кВт и 10 кВт выделяется на коробке, подшибниках и тягах — 10кВт печка разогреет до красна карданы и тяги к гранатам.
Отсюда вывод что такого быть не может. т.к. никакого тепла не выделяется, по факту. 0,5% не более т.к. подшибники и смазка есть — да нагрев идет незначительный и то больше от колодок чем от потерь на трении всех частей.

Отсюда вывод что стенд измеряет мощность в совокупности с инерционностью колес и шестеренок. Если знать все массы и размеры и построить формулу то все встанет на свои места, а когда масса колеса и всего остального неизвестна то остается догадываться что он там мериет. и почему разница мощности мотора и мощности с колес такая разная получается. — а по факту ее тупо подгоняют имея массу, размер колеса и массу размер вала.

Отсюда и типа литье поставил (легкие диски и машина сразу на 5 лошадок стала мощнее) откуда двигатель становится мощнее если прикрутили диски другие. как вес диска влияет на мощность мотора — никак.

По факту разгон быстрее машине веселее — ДА т.к. инерционность колеса становится меньше и раскрутить мотору проще 3 кг колесо чем колесо в 15 кг. и примерный расчет что 1 кг массы колеса равносилен 40 кг груза в авто. — так и получается. т.к. затраты на кинетическую энергию меньше.

Никто не заморачивается из производителей стендов т.к. всех устраивает то что есть и пару валов на палку насадить и поставить датчик с компом куда проще чем делать настоящую измерительную систему. которую не надо калибровать под каждый авто и которая будет мерить ту мощность которая есть.

Крутящий момент асинхронного электродвигателя

Крутящий момент электродвигателя – это сила вращения его вала. Именно момент вращения определяет мощность Вашего двигателя. Измеряется в ньютонах на метр или в килограмм-силах на метр.

Крутящий момент электродвигателя таблица

В данной таблице собраны крутящие моменты наиболее распространенных в Украине электродвигателей АИР, а также требуемый при пуске – пусковой, максимально допустимый для данного типа электродвигателя – максимальный крутящий момент и момент инерции двигателей АИР (усилие важное при подборе электромагнитного тормоза, например)

Номинальный

Номинальный — значение момента при стандартном режиме работы и стандартной номинальной нагрузке на двигатель.

Пусковой

Пусковой – это табличное значение. Сила вращения, которую в состоянии развивать электродвигатель при пуске.

При подборе эл двигателя убедитесь, что данный параметр выше, чем статический момент Вашего оборудования — насоса, либо вентилятора и т.д. В противном случае электродвигатель не сможет запуститься, что чревато перегревом и перегоранием обмотки.

Максимальный

Максимальный – предельное значение, по достижении которого нагрузка уравновесит двигатель и остановит его.

Расчет крутящего момента – формула

Примечание: при расчете стоит учесть коэффициент проскальзывания асинхронного двигателя. Номинальное количество оборотов двигателя не совпадает с реальным. Точное количество оборотов вы сможете найти, зная маркировку, в таблице выше.

Расчет онлайн

Для расчета крутящего момента электродвигателя онлайн введите значение мощности ЭД и реальную угловую скорость (количество оборотов в минуту)

тут будет калькулятор

После расчета крутящего момента, посмотрите схемы подключения асинхронных электродвигателей звездой и треугольником на сайте «Слобожанского завода»

Харьков, Полтавский шлях, 56, тел.: +38 (050) 775-43-34

© 2017 Слобожанский электромеханический завод. Все права защищены

Что Такое Крутящий Момент Двигателя Автомобиля



Большинство автовладельцев и водителей оценивают ходовые качества своих автотранспортных средств мощностью двигателя. В процессе эксплуатации транспортных средств часто возникают ситуации необходимости намеренного обгона сопутствующих машин в процессе движения. Находясь в определенном ритме движения, водитель «давит» на педаль акселератора и не получает желаемого ускорения обгона. В этом случае более информативной характеристикой приемистости двигателя является крутящий момент на определенных оборотах двигателя.

Максимальная мощность, указываемая в технических характеристиках двигателя, приводится на соответствующих оборотах. Для бензиновых ДВС обычно эта величина соответствует 5000 – 6000 оборотов в минуту, дизельных – приблизительно 3500 – 4500 об/мин. Поэтому считается, что все бензиновые движки являются высокооборотными, дизельные – низкооборотными. Это не всегда так.

Каждый автовладелец, особенно тот, который желает показать мастерство пилотирования симпатичным девушкам, должен знать характеристики крутящего момента своего авто.

Определение крутящего момента двигателя

Крутящий М момент силы согласно определению равен произведению F силы, действующей на рычаг L длиной. Формула, известная многим из школьного курса физики, представляет:

М=F*L

Если переводить входные величины в единую систему измерений, сила F измеряется в ньютонах, длина (в СИ) в метрах, М будет измеряться в ньютон на метр.

Сила, образуемая при воспламенении воздушно-топливной смеси, приводит в действие кривошипно-шатунный механизм. Чем больше рычаг, то есть разность расстояний от центра воздействия до места его осуществления, тем выше крутящий момент. Теоретически крутящий момент возможно пропорционально длине рычага увеличить. Но при этом уменьшится частота вращения двигателя, и увеличатся размеры механизма коленвала. В судах морских плаваний такие изменения несущественны, но автомобиль требует минимизации размеров всех комплектующих.

Крутящий момент ДВС определяет его мощность. Упрощенная формула для пересчета момента в параметр мощности имеет вид:

Р=М*n / 9549, где М – крутящий момента (в Н*м) на оборотах n (в об/мин). Р – мощность в киловаттах. 9549 – округленное число, полученное в результате сокращения констант.

Для пересчета мощности в более привычные для автолюбителей л.с. результат требуется умножить на 1,36.

Таким образом, мощность прямо пропорциональна количеству оборотов. В силу особенности конструкции бензиновые двигатели эффективно работают на оборотах до 8000 об/мин и выше. Таким образом, высокооборотные движки могут развить достаточно высокую мощность. У дизельных движков максимальная характеристика крутящего момента приходится на оборотах порядка 3500 – 4500 об/минуту. Обычно на таких оборотах происходит крейсерское движение автомобиля в городском ритме. Поэтому совершать маневры обгона и перестроения, резко увеличивая скорость на невысоких оборотах, на автомобилях с дизельными ДВС легче.

Характеристики момента приводятся в технических параметрах транспортного средства только вместе с величиной оборотов, для которых они измерены. В некоторых справочных данных автопроизводители указывают крутящий момент двигателя на холостых оборотах.

Наиболее полную картину ходовых параметров двигателя дают зависимости крутящего момента.

Зависимость мощности и крутящего момента двигателя

Крутящий момент по мере увеличения оборотов двигателя постепенно возрастает, при оборотах около 2800 немного стабилизируется, достигая своего максимума приблизительно 178 н*метр при 4500 об/мин. Мощность двигателя по мере увеличения оборотов продолжает возрастать, что согласуется с приведенной выше формулой. Однако после достижения величины оборотов 5400 об/мин, крутящий момент снижается с большей скоростью, чем растут обороты, и мощность уменьшается.

Это соответствует физической интерпретации процессов в двигателе. На малых оборотах в двигатель поступает мало топлива и воздуха, мощность невысокая. По мере увеличения оборотов сгорает больше топлива, вырабатывается больше энергии. При дальнейшем увеличении количества оборотов двигателя мощность начинает снижаться по причинам:

  • увеличение потерь на процессы трения;
  • кислородное голодание;
  • инерционные и другие механические потери;
  • тепловые потери.

Конструкторы ДВС стремятся расширить диапазон стабильного участка характеристики зависимости крутящего момента. В качестве одного из широко распространенных конструктивных решений применяются системы интеллектуального турбонаддува. Они позволяют избежать ситуации кислородного голодания на различных оборотах.

Крутящий момент относительно стабилен при оборотах двигателя от 2500 до 5500 об/мин. Водители могут смело начинать процесс обгона даже на малых оборотах.

Высокооборотные двигатели имеют стабильный момент до 6500 – 7500 об/мин. Это позволяет развить максимальную мощность на оборотах около 7500 об/мин, как приведено на рисунке 3.

Если вы подходите к покупке автомобиля серьезно, желательно покопаться в справочниках, на форумах, ознакомиться с дилерской информацией, погуглить, и найти зависимости крутящего момента и мощности. Тогда вы с научной точки зрения будете судить о ходовых параметрах автомобиля.

Выбирая автомобиль для эксплуатации в городских условиях, целесообразно приобрести дизельный авто, если вы любитель погонять с ветерком на автобанах, подойдет высокооборотный бензиновый двигатель.

Как увеличить крутящий момент

Характеристики крутящего момента двигателя формируются еще на этапе конструкторской разработки конкретной модели движка. Они также учитываются при расчетах тормозной системы, КПП, подвески и других систем. Самостоятельное увеличение крутящего момента двигателя может привести к преждевременному износу деталей авто.

Существует несколько способов повышения крутящего момента:

  • форсирование двигателя изменением параметров поршневой группы;
  • внесение изменений в топливную систему;
  • увеличение производительности воздухозабора;
  • чип-тюнинг.

Многие участники различных любительских автосостязаний используют комплексное форсирование двигателя. Однако следует помнить, что увеличение мощности и крутящего момента двигателя на четверть, уменьшает его ресурс вдвое.

Что такое мощность двигателя и крутящий момент. Как рассчитать мощность мотора

Автор Павел Александрович Белоусов На чтение 6 мин. Просмотров 561

Содержание

  1. Как рассчитывается мощность двигателя?
  2. Видео: Простыми словами без сложных формул и расчетов, что такое мощность, крутящий момент и обороты двигателя.
  3. Что такое крутящий момент
  4. Что лучше: мощность или крутящий момент

Мощность двигателя – это величина, показывающая, какую работу способен совершить мотор в единицу времени. То есть то количество энергии, которую двигатель передает на трансмиссию за определенный временной промежуток. Измеряется в киловаттах (кВт) или лошадиных силах (л. с.).

Как рассчитывается мощность двигателя?

Расчет мощности мотора проводится несколькими способами. Самый доступный способ – через крутящий момент. Умножаем крутящий момент на угловую скорость – получаем мощность двигателя.

N_дв=M∙ω=2∙π∙M∙n_дв

где:

N_дв – мощность двигателя, кВт;

M – крутящий момент, Нм;

ω – угловая скорость вращения коленчатого вала, рад/сек;

π – математическая постоянная, равная 3,14;

n_дв – частота вращения двигателя, мин-1.

Мощность рассчитывается и через среднее эффективное давление. Камера сгорания имеет определенный объем. Разогретые газы воздействуют на поршень в цилиндре с определенным давлением. Двигатель вращается с некоторой частотой. Произведение объема двигателя, среднего эффективного давления и частоты вращения, поделенное на 120, и даст теоретическую мощность двигателя в кВт.

N_дв=(V_дв∙P_эфф∙n_дв)/120

где:

V_дв – объем двигателя, см3;

P_эфф – эффективное давление в цилиндрах, МПа;

120 – коэффициент, применяемый для расчета мощности четырехтактного двигателя (у двухтактных ДВС этот коэффициент равен 60).

Для расчета лошадиных сил киловатты умножаем на 0,74.

N_(дв л.с.)=N_дв∙0,74

где:

N_дв л.с. – мощность двигателя в лошадиных силах, л. с.

Другие формулы мощности двигателя используются в реальных расчетах реже. Эти формулы включают в себя специфичные переменные. И чтобы измерить мощность двигателя по другим методикам, нужно знать производительность форсунок или массу потребленного двигателем воздуха.

На практике расчет мощности автопроизводители выполняют эмпирическим способом, то есть замеряют на стенде и строят график зависимости по факту, на основании полученных во время испытаний показателей.

Мощность двигателя – величина непостоянная. Для каждого мотора есть кривая, которая отображает на графике зависимость мощности от частоты вращения коленчатого вала. До определенного пика, примерно до 4-5 тысяч оборотов, мощность растет пропорционально оборотам. Далее идет плавное отставание роста мощности, кривая наклоняется. Примерно к 7-8 тысячам оборотов мощность идет на спад. Сказывается перекрытие клапанов на большой частоте вращения коленвала и падение КПД мотора из-за недостаточно интенсивного газообмена.

Чтобы узнать мощность двигателя, обратитесь к инструкции по эксплуатации авто. В разделе с техническими характеристиками мотора будет указана мощность и обороты, при которых она достигает пикового значения. Если мощность указана киловаттах, чтобы рассчитать лошадиные силы двигателя, воспользуйтесь приведенной выше формулой. В некоторых случаях автопроизводитель предоставляет график, на котором есть зависимость мощности двигателя и крутящего момента от частоты оборотов.

Видео: Простыми словами без сложных формул и расчетов, что такое мощность, крутящий момент и обороты двигателя.

Мощность ДВС определяет, насколько быстро автомобиль способен передвигаться или ускоряться (совершать работу). Полезная мощность двигателя рассчитывается с учетом потерь в трансмиссии, то есть указывает, сколько от изначальной мощности мотора по факту доходит до колес авто.

Что такое крутящий момент

Крутящий момент в двигателе автомобиля – это вращающая сила, которая численно равна произведению приложенной силы (давление раскаленных газов на поршень) на плечо (расстояние между осями коренных и шатунных шеек коленчатого вала в проекции, перпендикулярной оси вращения коленвала). Измеряется крутящий момент в ньютонах на метр (Нм).

Крутящий момент ДВС зависит от силы давления на поршень и расстояния между коренными и шатунными шейками. Зависимость здесь прямая. Чем больше плечо и чем больше давление на поршень – тем больше крутящий момент двигателя.

У дизельных двигателей степень сжатия больше. Больше и ход поршня в цилиндре (при равном с бензиновым мотором диаметре цилиндров). А это значит, что и расстояние между коренными и шатунными шейками будет больше. То есть длиннее плечо. За счет большей степени сжатия при рабочем такте у дизелей выше сила, давящая на поршень. Крутящий момент в дизельных моторах при прочих равных больше, чем в бензиновых.

Крутящий момент влияет на то, сколько энергии отдает мотор в текущий момент времени. Крутящий момент есть та величина, которая определяет фактически передаваемую в данный момент времени энергию на трансмиссию. Чем больше момент, тем сильнее тяга двигателя при текущих оборотах.

Что лучше: мощность или крутящий момент

Мощность и крутящий момент двигателя – величины взаимосвязанные. Это хорошо видно в формуле из первого пункта.

Пик крутящего момента на графике зависимости от частоты вращения мотора появляется раньше, чем пик мощности. Это справедливо как для дизельных, так и для бензиновых моторов. Однако у дизелей крутящий момент достигается раньше, и плато (интервал частоты вращения при пиковом значении) длиннее. У бензиновых ДВС мощность выше, хотя для ее достижения нужно раскрутить мотор почти до максимальных оборотов.

Сказать определенно, что лучше: мощность или крутящий момент, нельзя. Все зависит от случая. Трансмиссия современного авто способна трансформировать эти величины под требуемые условия. Поясним на примерах.

Для тяжелой техники, которой важна тяга в широком диапазоне оборотов, важнее крутящий момент. Мотор должен хорошо тянуть. Раскручивать его до предельных оборотов не нужно. Отчасти поэтому почти вся коммерческая техника оснащается дизельными моторами.

В гоночных автомобилях важнее мощность. Моторы этих авто по оборотам пилоты во время заездов держат в красной зоне. Двигатель отдает максимальную мощность. А трансмиссия преобразовывает мощность в тягу.

Для гражданских авто важен стиль вождения. Для езды на автомате подойдут оба мотора. Автоматическая трансмиссия будет держать мотор в диапазоне оборотов, при которых двигатель отдает максимум своего потенциала.

Для агрессивной езды на механике с раскручиванием двигателя в красную зону тахометра лучше подойдет бензиновый мотор. Но в этом случае нужно понимать, что для получения максимальной производительности от мотора потребуется держать его на пике оборотов и часто переключать передачи. Пик мощности у бензинового ДВС имеет малый диапазон и находится около максимальных оборотов. Для уверенных обгонов и ускорений нужно будет понижать передачу и раскручивать двигатель.

Для размеренной езды, особенно в городе, больше подходит дизель. Для обгона на дизельном авто зачастую не потребуется переходить на пониженную передачу, а высокий крутящий момент в широком диапазоне оборотов позволит реже переключаться.

Печать

Реставратор для пластика и кожи

5 минут и салон авто как новый. 
Посмотрите фото до и после

1490 р.

Набор для ремонта стекла

Ремонт стекла авто своими руками.
Спасает от трещин и сколов.

1690 р.

Зеркало видеорегистратор Vehicle Blackbox DVR

видеорегистратор + зеркало заднего вида + камера заднего вида
+ датчик движения + технология Dual cam + G-Sensor…

1990 р.

Зеркало — бортовой компьютер

12в1 — видеорегистратор, GPS-навигатор,
камера, интернет, радар, FM, G-sensor…

1990 р.

Авточехлы из экокожи

Салон будет как новый!
Легко чистятся, не трутся, не рвутся.

3990 р.

Фундаментальная проблема электронной мобильности

Внедрение электромобилей ускоряется из-за технологических улучшений, катализируемых государственной политикой и стимулами, направленными на достижение нулевого уровня выбросов. В 2020 году количество электромобилей в мире достигло 10 миллионов, но они составляют лишь 1% всех транспортных средств. По прогнозам, к 2030 году более четверти всех автомобилей, продаваемых в мире, будут электромобилями. Барьеры, стоящие перед внедрением электромобилей, в том числе инфраструктура зарядки, беспокойство по поводу запаса хода, а также стоимость и доступность моделей электромобилей, хорошо известны. Однако фундаментальная проблема, стоящая перед электрификацией мобильности, — это компромисс между скоростью и крутящим моментом.

Взаимосвязь крутящего момента и скорости — фундаментальная проблема электромобилей — легко описать, но оказалось трудно решить экономически эффективно. Транспортное средство должно уметь передвигаться, подниматься и спускаться с холмов, останавливаться на светофорах и разгоняться на автомагистралях, но при этом все это нужно делать за разумное время. Способность автомобиля приводить в движение колеса, подниматься в гору и ускоряться на шоссе зависит от крутящего момента. Насколько быстро он может двигаться, и время, необходимое для перемещения из одной точки в другую, зависит от скорости. Таким образом, фундаментальная проблема, которую должны решить все электромобили, заключается в том, какой крутящий момент и скорость необходимы для выполнения задач, для которых предназначен автомобиль. Это осложняется тем, что скорость и крутящий момент обратно пропорциональны. Это проблема соотношения крутящего момента и скорости.

Как определяются скорость и крутящий момент в электродвигателе

Скорость

Скорость двигателя определяется как скорость вращения двигателя и измеряется в оборотах в минуту или об/мин. Другими словами, скорость определяется как количество оборотов двигателя за 1 минуту.

Крутящий момент

Выходной крутящий момент двигателя представляет собой величину силы вращения, которую развивает двигатель, и измеряется в ньютон-метрах или Нм. Проще говоря, крутящий момент — это крутящая сила двигателя, но его может быть труднее понять, чем его эквивалентную скорость, поэтому давайте углубимся.

Требуемый крутящий момент определяется тремя факторами

1. Сопротивление качению

Сопротивление качению — это сила трения/противодействия, которую транспортное средство должно преодолевать в результате качения между колесами и поверхностью, по которой движется транспортное средство. Сопротивление качению зависит от материала шин и шероховатости поверхности. Чем выше сопротивление между шиной и дорогой, тем больше силы/крутящего момента необходимо двигателю для движения.

2. Сопротивление уклону

Сопротивление уклону — это сила тяжести, которая тянет автомобиль назад, когда он поднимается по наклонной поверхности. Чем круче подъем/холм, тем больше силы/крутящего момента требуется двигателю для подъема.

3. Сила ускорения

Сила ускорения помогает транспортному средству достичь заданной скорости из состояния покоя за определенный период времени. Крутящий момент двигателя имеет прямую зависимость от силы ускорения. Чем выше крутящий момент, тем меньше времени требуется автомобилю для достижения заданной скорости.

Суммарное тяговое усилие

Общее тяговое усилие — это общая сила, необходимая для перемещения транспортного средства с требуемыми показателями производительности, и представляет собой сумму трех сил, перечисленных выше. Таким образом, количество крутящего момента, которое должен развивать двигатель, определяется тем, насколько сильно транспортное средство имеет трение с дорогой, сколько требуется движения вверх и вниз по склону и как быстро водитель хотел бы перейти от нуля до полной скорости. .

Взаимосвязь крутящего момента и скорости: аналогия

Какой простой способ понять взаимосвязь крутящего момента и скорости? Представьте себе сценарий, в котором человека попросили забить длинный гвоздь в стену. Сколько раз человек ударяет гвоздем по головке за одну минуту, это скорость, насколько сильно каждый контакт с гвоздем, это крутящий момент. Однако у человека ограниченное количество энергии, и увеличение силы каждого удара или увеличение количества ударов в минуту приведет к более высокому потреблению энергии и заставит человека быстрее утомляться.

Теперь давайте определим две гипотетические ситуации, где в первом сценарии цель состоит в том, чтобы максимизировать количество ударов в минуту, а во втором сценарии цель состоит в том, чтобы как можно сильнее забить гвоздь один раз. В первом сценарии человек должен был бы поддерживать небольшое расстояние между гвоздем и молотком и, по сути, шевелить рукой, чтобы записать наибольшее количество ударов. Во втором сценарии человек должен был бы сделать полный ход, чтобы достичь пиковой силы.

Два сценария иллюстрируют обратную зависимость между скоростью и крутящим моментом, а также ограничения, которые существуют, когда целью является максимизация обоих. В зависимости от общей силы человека или номинальной мощности двигателя скорость и крутящий момент должны быть сбалансированы для достижения определенных целей.

Какая связь между крутящим моментом и скоростью?

В электродвигателе соотношение крутящего момента и скорости определяется по формуле: механическая мощность равна скорости, умноженной на крутящий момент . Соотношение крутящего момента и скорости обратно пропорционально, поскольку номинальная выходная мощность двигателя является фиксированной величиной. По мере увеличения выходной скорости доступный выходной крутящий момент пропорционально уменьшается. По мере увеличения выходного крутящего момента выходная скорость пропорционально уменьшается. Хотя скорость определяет максимальную скорость электродвигателя, больший крутящий момент позволяет системе достигать максимальной скорости за меньшее время. Динамические требования для предполагаемого применения будут определять, что лучше: больший крутящий момент или большая скорость.

Как производители электромобилей балансируют между скоростью и крутящим моментом?

Существует несколько распространенных способов, которыми сегодня производители электромобилей пытаются управлять соотношением крутящего момента и скорости.

Увеличенный размер двигателя

Увеличить номинальный размер двигателя означает выбрать более высокую номинальную мощность, чем это необходимо для обеспечения того, чтобы транспортное средство могло работать во всем диапазоне применений. Однако электродвигатели больших размеров имеют более высокую начальную стоимость, более высокие эксплуатационные расходы и приводят к потерям энергии.

Использование двигателя мощностью 400 кВт на полной мощности для приложения, которое может быть достигнуто с двигателем мощностью 200 кВт на полной мощности, даст хорошие результаты с точки зрения запуска приложения, но будет потреблять больше электроэнергии и, таким образом, быстрее разряжать батарею. сократить запас хода автомобиля и потребовать компонентов с более высокой номинальной мощностью, что приведет к дальнейшему увеличению затрат и снижению эффективности системы. Это один из современных способов, которым производители электромобилей управляют соотношением крутящего момента и скорости.

Несколько двигателей

Производители транспортных средств также могут увеличить выходную мощность системы, включив в нее несколько двигателей, чтобы расширить рабочий диапазон и сбалансировать соотношение скорости и крутящего момента. Производители электромобилей используют несколько двигателей для более высокой производительности, оптимальной механической компоновки и улучшенной тяги по более высокой цене. Компании традиционно выбирают двухдвигательную конфигурацию с асинхронным двигателем и двигателем с постоянными магнитами (ПМ). Двигатель с постоянными магнитами обычно используется транспортным средством для повышения дальности и эффективности, но асинхронный двигатель включается, когда транспортное средство должно быстро разогнаться. Преимущество асинхронного двигателя заключается в том, что он имеет минимальный крутящий момент для свободного хода, а это означает, что если он отстает и не используется, потери будут минимальными. Существуют также автомобили с 3-х и 4-х моторной конструкцией, которые предлагают превосходную производительность в дополнение к другим преимуществам, таким как лучшая управляемость и тяга.

Этот подход к обработке отношения крутящего момента и скорости также имеет свои недостатки. Несколько конструкций двигателей имеют свои оговорки. Дополнительные двигатели стоят дороже и увеличивают вес системы. Кроме того, необходимость в более крупной батарее, компонентах с более высокой номинальной мощностью и более высоком энергопотреблении нескольких двигателей еще больше увеличивает вес и стоимость системы. Комбинация приводит либо к уменьшению запаса хода, либо к увеличению общей стоимости владения.

Механическая коробка передач

Другим решением для достижения правильного баланса крутящего момента и скорости для данного применения является включение механической коробки передач. Механический редуктор — это устройство, используемое для увеличения выходного крутящего момента или изменения скорости двигателя. Редуктор состоит из ряда встроенных шестерен, которые изменяют крутящий момент и скорость между электродвигателем и нагрузкой в ​​зависимости от передаточных чисел.

Механический редуктор имеет длительный срок службы и поддерживает огромную силу; однако коробки передач имеют высокую стоимость и тяжелые по сравнению с другими компонентами автомобиля. Существует также потеря эффективности, когда мощность преобразуется механическими компонентами. Эти факторы приводят к снижению общей эффективности системы и увеличению стоимости дополнительных компонентов. Это еще один подход к управлению соотношением крутящего момента и скорости в электромобилях.

Принятие компромисса между крутящим моментом и скоростью

Другой альтернативой фундаментальной проблеме балансировки скорости и крутящего момента является принятие системных ограничений, когда речь идет о некоторых приложениях. Гоночный мотоцикл, например, предназначен для работы и, таким образом, может развивать невероятную скорость, но не может буксировать дом на колесах, поскольку у него нет необходимого крутящего момента для этого приложения. С другой стороны, полуприцеп оптимизирован для высокого крутящего момента и может тянуть чрезмерный вес, но не может развивать высокие скорости, которые может развить легковой автомобиль. Не имея лучших технологических решений, производители иногда просто принимают ограничения соотношения крутящего момента и скорости.

Новое решение для компромисса между крутящим моментом и скоростью: Coil Driver™

В технологии Coil Driver™ существует новый способ управления соотношением крутящего момента и скорости. Оснащенный передовой силовой электроникой, Coil Driver™ может управлять любым двигателем переменного тока с повышенной производительностью в широком диапазоне приложений вплоть до пиковой мощности и предлагает современное решение для компромисса между скоростью и крутящим моментом. Coil Driver от Exro — это новый динамический контроллер, который позволяет использовать несколько настроек мощности в одном двигателе. Этот контроллер заменяет стандартный контроллер электродвигателя и эффективно создает интеллектуальный электронный редуктор внутри двигателя. Это достигается за счет включения двух режимов работы в одном двигателе, один из которых оптимизирован для низкой скорости и высокого крутящего момента, а другой оптимизирован для высокой скорости и низкого крутящего момента. Coil Driver™ оптимизирует двигатель в режиме реального времени, плавно и по запросу, что позволяет оптимизировать эффективность для каждого режима работы, что приводит к более рациональному потреблению энергии, повышению производительности и снижению стоимости системы.

Exro Technologies также представила новое приложение для своей запатентованной технологии Coil Driver™, которая может значительно снизить стоимость и сложность, связанные с развертыванием инфраструктуры электромобилей в масштабе. Зарядное устройство Coil Drive убирает встроенное зарядное устройство, удаляет внешний выпрямитель переменного/постоянного тока, обеспечивает универсальную быструю зарядку переменного тока и добавляет возможности автомобиля ко всему (V2X).

Технологические достижения Exro Technologies не только решают фундаментальную проблему соотношения крутящего момента и скорости в электромобилях, но и устраняют барьеры, связанные с внедрением электромобилей.

Отношение крутящего момента к скорости в двигателе постоянного тока

спросил

Изменено 4 года, 5 месяцев назад

Просмотрено 89 тысяч раз

\$\начало группы\$

У меня есть концептуальное сомнение относительно отношения крутящего момента к скорости в двигателях постоянного тока. Вероятно, это пробел в моем мышлении, но я все равно задаю этот вопрос.

Говорят, что крутящий момент и скорость двигателя постоянного тока обратно пропорциональны. Но разве увеличение крутящего момента не приводит к увеличению углового ускорения и, следовательно, угловой скорости?

Я знаю, что обратная ЭДС/противоЭДС отвечает за обратное соотношение, но мне это кажется нелогичным. Что происходит с угловым ускорением, угловой скоростью при увеличении крутящего момента и куда уходит вся эта работа?

  • двигатель постоянного тока
  • скорость
  • крутящий момент
  • обратная ЭДС

\$\конечная группа\$

\$\начало группы\$

Концептуально вы должны думать об этом немного по-другому. То, как я думаю, вы думаете об этом, похоже на крутящий момент в автомобиле. Автомобиль с большим крутящим моментом будет быстрее разгоняться, что связано с увеличением скорости. Другими словами, вы нажимаете на педаль газа, чтобы увеличить скорость, и для этого вам нужен крутящий момент.

Однако, когда вы говорите о взаимосвязи между скоростью и крутящим моментом двигателя постоянного тока, вы должны думать об этом по-другому. Для данного двигателя с постоянным входным напряжением скорость двигателя будет определяться нагрузкой на валу двигателя. Для данной нагрузки единственный способ увеличить скорость — это увеличить напряжение. И это увеличение скорости потребует некоторого большего крутящего момента для ускорения, но после того, как он достигнет своей новой скорости, крутящий момент вернется к исходному крутящему моменту (если, конечно, нагрузка не зависит от скорости — как в вентиляторе).

Так что, может быть, вам лучше подумать об этом вместо того, чтобы говорить «Говорят, что крутящий момент и скорость в двигателе постоянного тока обратно пропорциональны», вы говорите « Для заданного напряжения , крутящий момент и скорость в двигателе постоянного тока говорят, что двигатель обратно пропорционален». Кривая скорость-крутящий момент, которую вы видите в техпаспорте, соответствует , только действительна для номинального напряжения, и двигатель будет работать по этой кривой. Таким образом, если крутящий момент увеличивается, скорость будет следовать этой кривой и падать.

\$\конечная группа\$

2

\$\начало группы\$

смоделируйте эту схему – Схема создана с помощью CircuitLab

Это стационарное приближение двигателя постоянного тока, который довольно хорошо работает с некоторыми типами двигателей постоянного тока (см. комментарий supercat). Поскольку стационарное состояние, индуктивность якоря \$L_a\$ не учитывается. У нас есть следующее: \$\$\begin{выравнивание} V &= \text{входное постоянное напряжение}\\ R_a &= \text{сопротивление якоря}\\ E_b &=\text{back-e.m.f}\\ \omega &= \text{угловая частота вращения вала} \ =\frac{2\pi \cdot \text{скорость}}{60}\\ I_a &= \text{ток якоря}\\ K_e &= \text{константа противо-ЭДС}\\ K_T &= \text{константа крутящего момента}\\ T &= \text{крутящий момент на валу} \end{выравнивание} \$\$ и применяются следующие уравнения: \$\$\begin{выравнивание} E_b &=K_e \cdot \omega . ..(1) \\ T &= K_T\cdot I_a …(2) \\ E_b &= V-I_a R_a …(3) (\text{получено из показанной эквивалентной схемы})\\ \text{из приведенных выше 3 уравнений} \\ T&=\frac{K_T V}{R_a}-\frac{K_e K_T \ \omega}{R_a} \end{выравнивание} \$\$ Уравнение, связывающее крутящий момент и скорость (или частоту), представлено ниже, ясно показывая, что крутящий момент обратно пропорционален скорости:

\$\конечная группа\$

4

\$\начало группы\$

Для постоянной мощности, подаваемой на механическую нагрузку, крутящий момент и скорость, умноженные вместе, являются константой. Это основное определение мощности, т.е.

Мощность =\$2\pi n T\$, где n — число оборотов в секунду, а T — крутящий момент.

Увеличение крутящего момента (и под увеличением крутящего момента я имею в виду угловую силу, возникающую при увеличении механической нагрузки) естественным образом вызывает замедление якоря, если мощность in постоянна.

Тем не менее, «двигатель постоянного тока» может означать что угодно, и многие двигатели будут иметь обмотки возбуждения, которые демонстрируют эффект типа «постоянной мощности», в то время как другие (с другими обмотками возбуждения) будут работать как регуляторы постоянной скорости и, таким образом, для увеличения крутящего момента ( из-за нагрузки), скорость остается почти постоянной.

Двигатели постоянного тока других типов могут иметь электронные контроллеры, которые делают то же самое; они воспринимают ток и по мере его увеличения увеличивают постоянное напряжение на якоре, что позволяет достичь почти постоянной скорости.

Я думаю, вы путаете реальный крутящий момент со способностью (или потенциальной) обеспечить реальный крутящий момент. Без механической нагрузки крутящий момент не имеет смысла, за исключением механических потерь в двигателе.

\$\конечная группа\$

\$\начало группы\$

Общее эмпирическое правило для щеточных машин постоянного тока:

Ток ~= Крутящий момент

Напряжение ~= (угловая) Скорость

(справедливости ради, почти все машины следуют этому правилу, но оно становится все менее и менее пропорциональным и больше «каким-то образом связано», например, частота)

У вас есть две константы (разновидности констант), когда речь идет об электрических машинах

Kt и Ke

Ke – константа напряжения на разомкнутой клемме с единицами измерения: Вольт/Вт. Это дает BackEMF

Kt – постоянная крутящего момента в единицах измерения: Nm/A

в теории Ke == Kt, но Kt зависит от характеристик железа (поэтому существует два).

Причина, по которой крутящий момент и скорость считаются обратно пропорциональными, заключается в том, что способность генерировать крутящий момент уменьшается с увеличением скорости.

Причина этого в том, что обратная ЭДС противодействует источнику питания, пытающемуся подать ток в статор, что создаст ЭМ-крутящий момент.

Вы правы в том, что для определенного приложения крутящего момента будет создаваться определенное ускорение, основанное на инерции ротора и инерции нагрузки, НО этот крутящий момент будет также уменьшаться с увеличением скорости (ветер, подшипники и т. д.). Таким образом, между уменьшающейся способностью нагнетать ток в машину при увеличении скорости, а также увеличением потерь при более высокой скорости скорость ускорения будет уменьшаться до тех пор, пока, наконец, не будет достигнута скорость холостого хода (или некоторая скорость под нагрузкой по сравнению с крутящим моментом нагрузки и генерируемым крутящим моментом). )

\$\конечная группа\$

\$\начало группы\$

Я согласен, что увеличение крутящего момента обязательно увеличит угловое ускорение, Но это не означает, что скорость всегда будет увеличиваться за счет увеличения крутящего момента или ускорения. Для увеличения скорости крутящий момент должен быть положительным (в направлении угловой скорости), а не обязательно увеличиваться. предположим, что угловая скорость = + 50 рад/сек. Пример1: крутящий момент1 = +5 Нм, крутящий момент2 = +10, увеличение скорости в обоих случаях. Пример 2: крутящий момент1 =+5 Н-м , крутящий момент2 = +3 скорость продолжает увеличиваться даже после уменьшения крутящего момента, но определенно с меньшей скоростью.

Eg3: крутящий момент1 = -5 Н-м , крутящий момент2 = -10 скорость уменьшается в обоих случаях . Например, 4: крутящий момент 1 = -5 Нм, крутящий момент 2 = -3 скорость продолжает снижаться даже после увеличения крутящего момента, но определенно с меньшей скоростью.

Во всех примерах предполагается, что угловая скорость положительна.

Так что думаю у вас сомнения именно в базовой динамике а не в машине.

\$\конечная группа\$

\$\начало группы\$

Любой ответ, который, кажется, отрицает, что ускорение пропорционально крутящему моменту или что мощность не пропорциональна скорости, просто чепуха. Здесь есть ответы, которые, кажется, отрицают это, но приукрашены изощренным (= ложной мудрости) языком. Итак, давайте будем откровенны. Если вы увеличиваете крутящий момент, вы увеличиваете скорость, ЕСЛИ вы не увеличиваете нагрузку. Важно помнить, что у электродвигателей есть обратная ЭДС (двигатель также является генератором), которая увеличивается со скоростью и ограничивает эффективное напряжение и, следовательно, ток и, следовательно, крутящий момент. Но больше крутящий момент? Больше скорости. Реф Исаак Ньютон.

\$\конечная группа\$

1

\$\начало группы\$

Я думаю, что для установившегося режима работы крутящий момент нагрузки должен быть равен крутящему моменту двигателя, теперь, если увеличить нагрузку для двигателя постоянного тока (шунт), ток возрастет. Теперь по-другому вы можете сказать, что если потребляемый ток увеличивается, крутящий момент двигателя увеличивается. Теперь по мере увеличения нагрузки скорость двигателя будет уменьшаться и будет стабилизироваться на скорости ниже, чем предыдущее значение, и, следовательно, вы можете сказать, что при увеличении нагрузки потребляемый ток увеличивается, но скорость уменьшается. Таким образом, вы можете сделать вывод, что по мере увеличения нагрузки (момент нагрузки) → крутящий момент двигателя увеличивается, но в то же время скорость уменьшается, поскольку крутящий момент нагрузки становится больше, чем крутящий момент двигателя.

\$\конечная группа\$

Твой ответ

Зарегистрируйтесь или войдите в систему

Зарегистрируйтесь с помощью Google

Зарегистрироваться через Facebook

Зарегистрируйтесь, используя электронную почту и пароль

Опубликовать как гость

Электронная почта

Требуется, но не отображается

Опубликовать как гость

Электронная почта

Требуется, но не отображается

Нажимая «Опубликовать свой ответ», вы соглашаетесь с нашими условиями обслуживания, политикой конфиденциальности и политикой использования файлов cookie

.

ток — Определение скорости и крутящего момента двигателя при заданной мощности и постоянном напряжении

\$\начало группы\$

Я пытаюсь определить, с какой скоростью будет работать мой щеточный двигатель постоянного тока при определенной рассеиваемой мощности. Предыстория: я построил токарный станок, используя этот двигатель, и для заданной глубины резания и скорости подачи (которая зависит от скорости двигателя и того, как быстро я поворачиваю рукоятку оси Z) требуется определенная мощность для удаления столько материала. Рассчитать мощность легко, если я сделаю предположение о частоте вращения двигателя, но если я запускаю двигатель на 12 В, я не имею прямого контроля над скоростью, только диапазон скоростей, верно?

Таким образом, если я установлю напряжение, а затем попытаюсь остановить двигатель (огромная глубина резания или очень высокая скорость подачи), я буду потреблять максимальный ток и иметь нулевую скорость, или я могу не выполнять резку и, по сути, нет скорости загрузки. Однако для данного разреза я могу найти мощность, но есть две точки на кривой крутящего момента и скорости (кроме максимальной мощности), которые дают эту мощность, так как я узнаю, в какой точке я нахожусь? Точка низкого крутящего момента с высоким крутящим моментом или точка низкого крутящего момента с высокой скоростью?

Спасибо!

  • мощность
  • ток
  • двигатель постоянного тока
  • скорость
  • крутящий момент

\$\конечная группа\$

5

\$\начало группы\$

Глядя на техническое описание, хотя оно и не лжет, оно определенно вводит в заблуждение.

Обратите внимание, что заявленная «максимальная мощность» достигается почти точно при половине оборотов двигателя без нагрузки и при половине тока останова. Это действительно точка «максимальной мощности» для такого двигателя, но в техническом описании не упоминается, что это также номинальная точка КПД 50%, таким образом, рассеивание 12 В * 68 А-337 Вт = 479. W в этом маленьком моторчике — уничтожит его, вероятно, за считанные минуты.

(В идеале должна отдаваться ровно половина мощности, около 400 Вт на валу и 400 Вт на нагреве, но двигатель не идеален).

Двигатель, вероятно, подходит для непрерывной мощности 100-150 Вт и кратковременной мощности 200-250 Вт.

Таким образом, практически вы должны управлять двигателем в верхней части диапазона скоростей, и если скорость падает ниже (скажем) 70% от скорости без нагрузки (или ток возрастает до 30% от тока остановки), то — если это не является строго временным, например, запуск тяжелой нагрузки или попадание в замерзшее место при обработке чугунной поверхности, вам необходимо отключить ток и защитить двигатель.

Тогда вопрос о том, какая сторона кривой крутящего момента и скорости не применяется — если не сработала защита, вы должны быть на стороне высокой скорости.

Вы можете приобрести автоматические выключатели, которые допускают кратковременную перегрузку по току. Это автоматические выключатели класса C для двигателей переменного тока, используемых в большинстве станков. Однако я не знаю ничего подходящего для 12 В постоянного тока. Я бы искал источник постоянного тока 12 В, который можно настроить на отключение, если его выход превышает 40 А более чем на пару секунд. И, как говорит Олин, если вы хотите контролировать это самостоятельно, измерение тока, безусловно, лучший способ.

\$\конечная группа\$

\$\начало группы\$

Можете ли вы определить и отфильтровать скачки напряжения или тока при включении двигателя, чтобы вызвать однократный выстрел, а затем сделать вывод скорости тахометра для сервоуправления.

Или ограничить текущие импульсы, LPF и вход для PLL типа II и контролировать управление VCO, напряжение для скорости тахометра?

1-й вариант лучше, если вы можете обнаружить обратную ЭДС с помощью чистого импульсного преобразователя.

\$\конечная группа\$

3

\$\начало группы\$

Вероятно, самый простой способ определить рабочую точку двигателя — включить последовательно с ним амперметр. Вы можете заранее измерить ток останова и ток холостого хода. Любой фактический рабочий ток должен находиться в пределах этого диапазона.

Поскольку у вас видимо источник фиксированного напряжения (12 В в вашем случае), мощность прямо пропорциональна току. Фактически, мощность в ваттах — это ток в амперах, умноженный на напряжение в вольтах. Обратите внимание, однако, что это мощность, поступающая в двигатель. Выходная механическая мощность будет разной. Один из очевидных способов увидеть это — рассмотреть входную и выходную мощности при остановленном двигателе. Электрическая входная мощность будет максимальной, а механическая выходная мощность будет равна нулю. В этом случае вся мощность уходит на нагрев двигателя, который не может быть рассчитан на неопределенный срок.

\$\конечная группа\$

1

\$\начало группы\$

Хотя для данного разреза я могу найти мощность, но есть две точки по кривой крутящего момента (кроме максимальной мощности), которые дают это власть, так как я знаю, в какой точке я нахожусь? Низкая скорость с высоким крутящим моментом точка или точка высокой скорости с низким крутящим моментом?

По дыму видно 🙂

В точке низкого крутящего момента на высоких оборотах двигатель должен работать относительно эффективно (выдавая больше мощности, чем рассеивается), но в точке высокого крутящего момента на низких оборотах происходит обратное. Большинство моторов , а не , предназначены для рассеивания гораздо большей мощности, чем они выдают, поэтому их работа в этом регионе вызовет их перегрев.

Двигатель постоянного тока с постоянными магнитами или с параллельным возбуждением имеет линейную зависимость крутящего момента от тока и обратнозависимую скорость вращения от крутящего момента при изменении нагрузки, обеспечивая максимальную выходную мощность примерно при 50% оборотов холостого хода.

Ваш двигатель имеет следующие характеристики:

 Напряжение: 12 В постоянного тока
Без нагрузки RPM: 5310 (+/- 10%)
Свободный ток: 2,7 ампер
Максимальная мощность: 337 Вт (при 2655 об/мин, 172 унции на дюйм и 68 А)
Крутящий момент: 2,42 Нм или 343,4 унции на дюйм
Ток срыва: 133 ампера
 

При максимальной выходной мощности он работает ровно на 50 % оборотов в минуту, производя механическую мощность 337 Вт из потребляемой электроэнергии 816 Вт при КПД 41 %. В этой рабочей точке он должен рассеивать 479 Вт, что, вероятно, уже превышает его постоянную мощность. Любая более высокая нагрузка быстро увеличивает рассеиваемую мощность, до 1596 Вт в режиме ожидания! Вы не хотите эксплуатировать двигатель в этом регионе.

\$\конечная группа\$

Твой ответ

Зарегистрируйтесь или войдите в систему

Зарегистрируйтесь с помощью Google

Зарегистрироваться через Facebook

Зарегистрируйтесь, используя электронную почту и пароль

Опубликовать как гость

Электронная почта

Требуется, но не отображается

Опубликовать как гость

Электронная почта

Требуется, но не отображается

Нажимая «Опубликовать свой ответ», вы соглашаетесь с нашими условиями обслуживания, политикой конфиденциальности и политикой использования файлов cookie

.

Связь между крутящим моментом и скоростью

В физике вы часто слышите слово крутящий момент. Вы точно знаете, что такое крутящий момент? Ну, крутящий момент — это не что иное, как сила, приложенная к объекту, чтобы заставить его вращаться вокруг своей оси. Следовательно, любая сила, которая может вызвать это угловое ускорение в объекте, является крутящим моментом.

 

Как вы, вероятно, поняли из этого определения, крутящий момент является векторной величиной, поскольку здесь действуют как величина, так и направление силы. Направление вектора крутящего момента зависит от направления силы на оси.

 

Общий вопрос, который может возникнуть при изучении крутящего момента, касается его отношения к скорости. Поэтому ниже приведено уравнение, которое связывает крутящий момент и скорость.

 

\[\text{Крутящий момент}(\tau) = \frac{Мощность}{скорость}\]

 

Это самая основная форма зависимости между крутящим моментом и скоростью. Если вы хотите узнать больше, вы должны сначала определить, что такое скорость.

 

Быстрое упражнение — 1

В. Колесо движется со скоростью 0,3 м/с при приложении мощности 40 Вт. Определите крутящий момент, действующий на колесо, используя уравнение зависимости крутящий момент-скорость.

Решение –

Скорость колеса 0,3 м/с

Мощность на колесе 40 Вт

Таким образом, \[Крутящий момент = \frac{Мощность}{скорость}\]

\[Крутящий момент = \frac{40}{0,3} \]

Крутящий момент равен 133,33 ньютон-метра.

 

Что такое скорость?

Чтобы правильно оценить крутящий момент в зависимости от скорости, вы должны сначала понять скорость в деталях. Скорость есть не что иное, как расстояние, пройденное объектом за единицу времени. Скорость является скалярной величиной. Вам не нужно устанавливать направление движения, чтобы определить скорость тела.

 

Это также то, что отличает скорость от скорости. Будучи векторной величиной, скорость – это скорость тела в определенном направлении.

 

Формула крутящего момента и скорости

Связь между этими двумя величинами и их формулой можно легко понять с помощью формулы для расчета мощности, переносимой объектом, движущимся по кругу.

 

\[Мощность = Крутящий момент \умножить на скорость\]

 

\[P = \tau \times \omega \]

 

Где p представляет мощность или работу, совершаемую объектом при круговом движении . T — крутящий момент (крутящий момент считается вращательной способностью тела, рассматриваемой как эквивалент силы), а \[\omega\] — угловая скорость или скорость, достигаемая движущимся объектом (рассматривается как скорость изменения углового смещение).

 

Приведенное выше уравнение можно преобразовать, чтобы получить формулу для определения крутящего момента движущегося объекта для расчета скорости (угловая скорость/скорость)

 

\[\omega= \frac{P}{T} \]

 

Сила здесь обычно измеряется в ваттах (Вт) или лошадиных силах (л. с.) . В двигателях это в основном механическая выходная мощность двигателя. Для электродвигателей скорость измеряется в оборотах в минуту или в об/мин. Она зависит от скорости вращения движущейся части. Крутящий момент для электроприборов измеряется либо в дюймо-фунтах (в фунтах), либо в ньютон-метрах (Н·м) и определяет силу, действующую на двигатель или другой объект, совершающий круговое движение. Это сила вращения, которую развивал объект.

 

Вывод формулы соотношения между крутящим моментом и скоростью

Поскольку крутящий момент представляет собой вращательное движение, мы можем легко вывести его отношение к мощности, сравнив линейный эквивалент. Чтобы определить линейное перемещение, просто умножьте радиус перемещения на покрываемый угол. Имейте в виду, что линейное перемещение относится к расстоянию, проходимому по окружности колеса.

 

Следовательно, мы можем сказать, что \[\text{Линейное расстояние} = Время \times \text{Угловая скорость} \times Радиус\] (уравнение 1) 

 

Мы знаем, что \[Момент = Сила \умножить на Радиус \]

 

\[Сила = \frac{Момент} {Радиус}\] (уравнение 2)

 

Теперь \[Мощность = \frac{Force \times \text{Linear Distance}}{Time} \]

 

Интегрируя значение силы из уравнений 1 и 2, получаем

 

\[Power = \frac{( \frac{Крутящий момент}{Радиус}) \times Время \times \text{Угловая скорость} \times Радиус}{Время} \]

 

Таким образом, \[Мощность = Крутящий момент \times \text{Угловая скорость}\]

 

Следовательно, \[Крутящий момент = \frac{Мощность}{\text{Угловая скорость}}\]

 

Какая связь между крутящим моментом и скоростью?

Математическая формула говорит нам, что эта сила вокруг оси обратно пропорциональна скорости (угловой скорости). Это означает, что увеличение скорости приводит к падению крутящего момента и наоборот.

 

Еще один важный фактор, который необходимо учитывать, заключается в том, что в этом уравнении скорость и скорость взаимозаменяемы. Это связано с тем, что крутящий момент, будучи векторной величиной, всегда будет иметь скорость в определенном направлении. Теперь, когда вы знаете соотношение крутящего момента и скорости, ответьте на этот простой вопрос.

 

Верно или неверно – 1

В. Крутящий момент прямо пропорционален радиусу вращения.

Ответ. Истинный. Поскольку крутящий момент является произведением силы и радиуса вращения, увеличение этого радиуса также увеличивает результирующий крутящий момент. То же верно и для противоположного.

 

Связь между крутящим моментом и скоростью в двигателе постоянного тока

В двигателе постоянного тока скорость рассчитывается как число оборотов в минуту. Таким образом, для такого двигателя можно определить крутящий момент по следующей формуле –

 

\[Крутящий момент = \frac{Мощность}{(2\pi \times \text{Скорость вращения})}\]

 

Наши онлайн-курсы и широкий выбор книг в формате PDF помогут вам в дальнейшем понимание зависимости крутящего момента от скорости. У нас также есть сеансы устранения сомнений, чтобы обеспечить правильное понимание каждой темы. Теперь вы даже можете загрузить наше приложение Vedantu, чтобы с легкостью получить доступ к онлайн-сессиям.

Характеристики момента-скольжения и момента-скорости асинхронного двигателя

Двигатели

Электрические технологии

2 4 минуты чтения

Характеристики крутящего момента-скорости и скольжения асинхронного двигателя

Содержание

Характеристики крутящего момента-скольжения

Характеристики крутящего момента-скольжения дают взаимосвязь между крутящим моментом и скольжением. Характеристики проскальзывания крутящего момента показывают, как изменяется крутящий момент при изменении проскальзывания. Скольжение определяется как отношение синхронной скорости к фактической скорости ротора. Фактическая скорость ротора зависит от условий нагрузки. Следовательно, скольжение изменяется в зависимости от условий нагрузки.

В предыдущей статье мы вывели уравнение крутящего момента асинхронного двигателя.

Из приведенного выше уравнения, если R 2 и X 20 остаются постоянными, крутящий момент зависит от скольжения. Кривая характеристики момента-пробуксовки выглядит как прямоугольная гипербола. И эта кривая делится на три области;

  • Область низкого скольжения
  • Область среднего скольжения
  • Область высокого скольжения

Кривая характеристики момента-пробуксовки для различных значений сопротивления ротора показана на рисунке ниже.

 

Связанный пост:

  • Уравнение крутящего момента асинхронного двигателя
  • Эквивалентная схема асинхронного двигателя
Низкий Область скольжения

При синхронной скорости скольжение асинхронного двигателя равно нулю. Следовательно, вращающий момент, развиваемый в роторе, равен нулю. Поэтому асинхронный двигатель всегда работает немного меньше синхронной скорости. И в этом состоянии проскальзывание очень низкое.

Когда проскальзывание очень низкое, (sX 20 ) 2 незначительно по сравнению с R 2 . Итак, для условий малого скольжения

Если мы считаем сопротивление ротора R 2 постоянным;

T ∝ s

Следовательно, в условиях малого скольжения крутящий момент прямо пропорционален скольжению. Это нормальная рабочая область для асинхронного двигателя. В области малого скольжения кривая крутящего момента представляет собой прямую линию.

Область среднего скольжения

При увеличении нагрузки скорость асинхронного двигателя снижается, а скольжение увеличивается. По мере увеличения скольжения член (sX 20 ) 2 становится большим по сравнению с сопротивлением ротора R 2 . И в этом условии можно пренебречь сопротивлением ротора R 2 .

Итак, крутящий момент обратно пропорционален скольжению. В этой области кривая образует прямоугольную гиперболу и проходит через точку максимального крутящего момента. Максимальный крутящий момент достигается при R 2 = сХ 20 . Этот крутящий момент известен как момент отрыва или крутящий момент опрокидывания.

Область высокого скольжения

Если мы увеличим крутящий момент выше точки максимального крутящего момента, крутящий момент начнет уменьшаться. Это состояние при увеличении нагрузки. В этом состоянии скорость двигателя снижается, и необходимо активировать защиту от перегрузки, чтобы отключить двигатель от источника питания. Если двигатель постоянно работает в этой области, двигатель выйдет из строя из-за перегрева. Эта область на кривой крутящего момента-проскальзывания является убывающей областью после точки максимального крутящего момента.

Как правило, асинхронный двигатель работает при значении скольжения от нуля до S M . Проскальзывание S M — это проскальзывание в точке максимального крутящего момента. Момент выдергивания асинхронного двигателя в 2–3 раза превышает номинальный момент при полной нагрузке для типичной работы. Таким образом, двигатель может выдерживать кратковременную перегрузку без остановки.

Кривая проскальзывания момента асинхронного двигателя при постоянном сопротивлении ротора показана на рисунке ниже.

Как и в предыдущем описании, эта кривая также разделена на три части;

  • Автомобильный регион
  • Генерирующая область
  • Область разрыва
Автомобильный регион

В этом режиме работы скольжение асинхронного двигателя составляет от нуля до единицы. Когда на статор подается электроэнергия, ротор вращается ниже синхронной скорости. И крутящий момент двигателя изменяется от нуля до крутящего момента при полной нагрузке, а скольжение изменяется от нуля до единицы.

В этом состоянии крутящий момент прямо пропорционален скольжению. Как правило, в этой области работает асинхронный двигатель. Проскальзывание равно нулю на синхронной скорости и равно единице в состоянии покоя.

Область генерации

В режиме генерации асинхронный двигатель работает со скоростью, превышающей синхронную, и ведет себя как асинхронный генератор. Скорость двигателя увеличивается выше синхронной скорости с помощью внешних устройств, таких как первичный двигатель.

В области генерации и скольжение, и крутящий момент отрицательны. Следовательно, машины получают механическую энергию и отдают электрическую энергию. Во время генерации двигатель должен подавать реактивную электроэнергию.

Область торможения

В области торможения меняется полярность напряжения питания. Следовательно, двигатель вращается в обратном направлении. Этот режим используется для остановки двигателя. Этот метод электрического торможения известен как подключение. В режиме торможения скольжение больше единицы.

При использовании этого метода двигатель останавливается за короткое время. Но кинетическая энергия, запасенная в нагрузке, рассеивается в виде тепла. Поэтому во время торможения выделяется очень большое количество тепла. А также, если статор подключен к источнику питания, он также вырабатывается в виде тепла. Следовательно, необходимо отключить питание статора перед входом в режим торможения.

  • Запись по теме: Уравнение мощности, напряжения и ЭДС двигателя постоянного тока

Характеристики крутящий момент-скорость асинхронного двигателя

Характеристика крутящий момент-скорость представляет собой кривую между крутящим моментом и скоростью асинхронного двигателя и показана на рисунке ниже.

Приведенное ниже уравнение дает скорость вращения ротора, при которой достигается максимальный крутящий момент.

Н М = N S (1 -S M )

Уравнение максимального крутящего момента:

Здесь мы видим, что максимальный крутящий момент не зависит от сопротивления ротора. Но точное расположение максимального крутящего момента зависит от сопротивления ротора. Чем больше значение сопротивления ротора, тем больше значение скольжения, при котором достигается максимальный крутящий момент.

Из кривой момент-скорость видно, что по мере увеличения сопротивления ротора скорость отрыва двигателя уменьшается (максимальный момент остается постоянным).

Related Posts:

  • Типы электродвигателей – классификация двигателей переменного и постоянного тока и специальных двигателей
  • Применение электродвигателей
  • Однофазный асинхронный двигатель – конструкция, работа, типы и применение
  • Трехфазный асинхронный двигатель – конструкция, работа, типы и применение
  • Пускатель двигателя – Типы пускателя двигателя и методы пуска двигателя
  • Пускатель прямого действия — схема подключения пускателя DOL для двигателей
  • Расчет размера кабеля для двигателей LT и HT
  • Методы управления скоростью двигателя постоянного тока
  • Машина постоянного тока – конструкция, работа, типы и применение
  • Серводвигатель – типы, конструкция, работа, управление и применение
  • Бесщеточный двигатель постоянного тока (BLDC) – конструкция, принцип работы и применение
  • Шаговый двигатель
  • – типы, конструкция, работа и применение
  • Что такое КПД двигателя и как его повысить?
  • Асинхронный двигатель и линейные асинхронные двигатели Формулы и уравнения
  • Что такое мотор-генераторная установка и как она работает?
  • Как запустить трехфазный асинхронный двигатель от однофазного источника питания?
  • Формулы и уравнения в области электротехники и электроники
  • Символы электродвигателей

Показать полную статью

Связанные статьи

Кнопка «Вернуться к началу»

Крутящий момент и угловое ускорение: определения и взаимосвязь

В поступательной динамике крутящий момент или момент — это векторная величина, необходимая для вращения объекта вокруг точки поворота. Математически это произведение силы, действующей на точку, и перпендикулярного расстояния от силы до точки поворота. Крутящий момент измеряется в ньютон-метрах, как показано в приведенном ниже уравнении, где F — сила, а d — перпендикулярное расстояние.

T=F[N]·d[m]

Принцип моментов в поступательной динамике гласит, что когда объект находится в равновесии, сумма моментов по часовой стрелке относительно точки поворота равна сумме моментов против часовой стрелки относительно точки поворота. поворотный момент. Так где же возникает угловое ускорение? Давай выясним!

Крутящий момент и угловое ускорение: Значение

Крутящий момент также может быть выражен в терминах динамики вращения, где он необходим для создания углового ускорения объекта, заставляющего объект вращаться. Поступательный крутящий момент представляет собой произведение перпендикулярного расстояния и касательной силы, действующей на точку. Крутящий момент вращающегося объекта, показанный на рисунке 1, можно найти, используя второй закон Ньютона и линейный крутящий момент.

Рис. 1. Крутящий момент и угловое ускорение в динамике вращения. Источник: Джорджия Панаги, StudySmarter.

Выразим крутящий момент T через поступательную силу F t в ньютонах и радиус r в метрах. Затем применим второй закон Ньютона. В показанном выше объекте мы получаем выражение для поступательной силы через массу и ускорение, где m — масса, а a — ускорение в м/с 2 .

T=Ft·r·sinθГдеsinθ=1asθ=90०F=ma⇒Ft=mat

Мы используем поступательное ускорение a t в связи с полученным уравнением, которое выражает поступательное ускорение через радиус, и мы подставьте его в уравнение поступательной силы.

at=a·rFt=m·(a·r)

Затем мы используем приведенную выше формулу крутящего момента, но подставляем полученное выражение поступательной силы. Для угла 90 градусов sinθ равен единице. Следовательно, крутящий момент равен произведению массы, ускорения и квадрата радиуса, как показано ниже:

T=(mar)·r·sinθ=m·r2·a

Полученное выражение можно записать с точки зрения момента инерции I, измеряемого в кгм 2 , умноженного на ускорение, измеряемое в м/с 2 , так как момент инерции равен массе объекта, умноженной на квадрат расстояния до оси вращения.

T=(m·r2)·aT=I·a

Какова формула крутящего момента для динамики вращения?

Как было выведено выше, крутящий момент для вращательного движения определяется как произведение момента инерции и углового ускорения, как показано ниже, где T — крутящий момент, измеренный в ньютон-метрах. I — это момент инерции, который представляет собой тенденцию вращающегося объекта сопротивляться угловому ускорению. Угловое ускорение α – скорость изменения угловой скорости.

T[Нм]=I[кгм2]·a[рад/с2]

где I[кгм2]=m·r2

Из формулы можно вывести, что крутящий момент зависит от момента инерции и, следовательно, от распределение массы объекта и его расстояние до центра вращения. Угловое ускорение пропорционально величине крутящего момента.

Момент инерции является вращательным эквивалентом силы вокруг оси в поступательной динамике, а угловое перемещение эквивалентно линейному перемещению во вращательной динамике.

Крутящий момент также равен скорости изменения углового момента, если масса сохраняется. Это выведено ниже, где ω — угловая скорость в рад / с, а L — угловой момент. Мы используем предыдущую формулу крутящего момента через момент инерции и заменяем ускорение скоростью изменения угловой скорости. Это дает нам скорость изменения углового момента, которая равна крутящему моменту, как показано ниже.

T[Nm]=I·aT=I·dωdtL=I·ωT=dLdt

Если на объект действуют две или более сил, общий крутящий момент представляет собой векторную сумму крутящих моментов, как показано ниже, где n число крутящие моменты присутствуют.

∑T=T1+T2+…+Tn

Направление крутящего момента

Поскольку крутящий момент является векторной величиной, для его определения необходимы как величина, так и направление. Направление крутящего момента можно найти с помощью правила правой руки, где четыре пальца правой руки указывают в направлении приложенной силы F. Направление крутящего момента совпадает с направлением большого пальца.

Рис. 2. Направление крутящего момента, когда сила направлена ​​вверх. Источник: Джорджия Панаги, StudySmarter.

Пример показан на рис. 2, где сила приложена вверх, а результирующий крутящий момент показан ниже. Если приложенная сила направлена ​​вниз, результирующий крутящий момент направлен в противоположном направлении, как показано на рис. 3 .

Рис. 3. Направление крутящего момента, когда сила направлена ​​вниз. Источник: Джорджия Панаги, StudySmarter.

Если направление крутящего момента относительно точки вращения по часовой стрелке, крутящий момент можно считать отрицательным, и наоборот, поскольку углы в направление против часовой стрелки считается положительным.

Колесо вращается вокруг оси А. Касательная сила 30 Н приложена к краю колеса радиусом 40 см. Колесо линейно ускоряется из состояния покоя, достигая скорости вращения 10 рад/сек за 8 сек. Определить момент инерции колеса.

Начнем с уравнения крутящего момента.

T=F·r=30N·0,4m=12NmT=I·a⇒I=12/a

Для определения углового ускорения в рад/с 2 , нам нужно найти скорость изменения угловой скорости.

a=dωdt=10-08с-0с=1,25рад/с2

Затем мы подставляем его в предыдущее уравнение, чтобы найти момент инерции:

I=12Нм1,25рад/с2=9,6кгм2 показано ниже на рис. 4. Определить чистый крутящий момент колеса относительно его центра и угловое ускорение колеса, если момент инерции равен 20 кгм 2 . R 1 — 5 см, R 2 — 12 см, F 1 — 15 Н (желтый цвет на рисунке ниже) и F 2 – 18 Н (на рисунке синий).

Мы создаем диаграмму для визуализации проблемы.

Рис. 4. Пример чистого крутящего момента. Источник: Джорджия Панаги, StudySmarter.

Общий крутящий момент представляет собой векторную сумму отдельных крутящих моментов, действующих на объект.